Anda di halaman 1dari 84

Copyright 2014 Delhi Academy of Medical Sciences, All Rights Reserved.

1/84
Test Information
Test Name SWTS-PHARMACOLOGY Total Questions 200

Test Type Examination Difficulty Level Difficult

Total Marks 600 Duration 120minutes

Test Question Language:- ENGLISH

(1). All of the following have inotropic action, except

a. Dopamine

b. Amrinone

c. Amiodarone

d. Isoprenaline

Solution. Ans-1: (c) Amiodarone


Ref: Read the text below
Sol :
* Amiodarone, an antiarrhythmic, inhibits myocardial contractility.
* Dopamine (also beta 1 agonist), amrinone (inotropic + vasodilator = inodilator) and isoprenaline (beta agonist) have positive inotropic
effect, i.e. myocardial contractility.

Your Answer. c
Correct Answer. c

(2). Which of the following antimicrobials needs dose reduction even in mild renal failure

a. Ciprofloxacin

b. Carbenicillin

c. Cefotaxime

d. Ethambutol

Solution. Ans-2: (d) Ethambutol


Ref: Read the text below
Sol:
* Aminoglycosids, Amphotericin B, cephalosporins, vancomycin, flucytosine and ethambutol require dose reduction even in mild renal
failure.

Your Answer. c
Correct Answer. d

Copyright 2014 Delhi Academy of Medical Sciences, All Rights Reserved. 2/84
(3). The phenomenon of antibiosis was first demonstrated by

a. Gerhard Domagk

b. Louis Pasteur

c. Alexander Fleming

d. Selman Waksman

Solution. Ans-3: (b) Louis Pasteur


Ref.: Read the text below
Sol :
Louis Pasteur. He showed that growth of anthrax bacilli in urine can be inhibited by exposure to air-borne bacteria.
* Gerhard Domagk used prontosil for treatment of streptococcal infections. Sulphonamides were later developed from these dyes.
* Alexander Fleming discovered penicillin.
* Selman Waksman isolated streptomycin.

Your Answer. c
Correct Answer. b

(4). All of the following have been shown to be beneficial in patients with congestive heart failure except

a. Enoximone

b. Ivabradine

c. Thalidomide

d. Vernakalant

Solution. Ans-4: (d) Vernakalant


Ref.: Read the text below
Sol :
Vernakalant. It is an antiarrhythmic.
* Enoximone is a recent PDE3 inhibitor.
* Ivabradine decreases death rate and risk of hospitalization d/t HF when added to conventional treatment. It can be used in systolic
dysfunction with normal sinus rhythm and HR 70 bpm.
* Thalidomide has been found to increase LVEF in some cases.

Your Answer. c
Correct Answer. d

(5). Which of the following is most commonly responsible for causing superinfections?

a. Clindamycin

b. Tetracyclines

c. Cefoperazone

d. Piperacillin

Solution. Ans-5: (c) Cefoperazone


Ref.: Read the text below
Sol :
2nd and 3rd generation cephalosporins are the most common cause of superinfections in recent times.

Your Answer. a
Correct Answer. c

Copyright 2014 Delhi Academy of Medical Sciences, All Rights Reserved. 3/84
(6). Which of the following acts as a predominant arteriolar dilator?

a. Nitroglycerine

b. Clevidipine

c. Enalaprilat

d. Nitroprusside

Solution. Ans-6: (b) Clevidipine


Ref.: Read the text below
Sol :
Clevidipine.
* Predominant venodilators: Nitrates.
* Predominant arteriolar dilators: CCBs, KCOs and hydralazine.
* Rest are mixed vasodilators.

Your Answer. d
Correct Answer. b

(7). Pneumococcal resistance to penicillin G is mainly acquired by

a. Conjugation

b. Transduction

c. Transformation

d. All of the above

Solution. Ans-7: (c) Transformation


Ref: Read the text below
Sol:
* Acquisition of antibiotic resistance by Transduction is common in Staphylococcal and that of by Transformation in Pneumococcus and
Neisseria.
* Vancomycin resistance in enterococci and staphylococcus is mediated by conjugative plasmid.

Your Answer. b
Correct Answer. c

(8). Maintenance dose of digoxin in a child as mg/kg is

a. 0.001 0.005

b. 0.01 0.02

c. 0.04 0.06

d. 0.06 0.08

Solution. Ans-8: (b) 0.01 0.02


Ref: Read the text below
Sol :
* Maintenance dose of digoxin in children is 0.01-0.02 mg/kg/day.
* It is a little higher than that in adults (about 0.005 mg/kg/day).
* Dose depends on lean body mass and creatinine clearance of the body.

Your Answer. a
Correct Answer. b

Copyright 2014 Delhi Academy of Medical Sciences, All Rights Reserved. 4/84
(9). Anticoagulant action both in-vitro and in-vivo is seen with

a. Coumarin

b. Heparin

c. E.D.T.A.

d. Phenindione

Solution. Ans-9: (b) Heparin


Ref: Read the text below
Sol :
* Heparin is the anticoagulant, can be used in-vivo as well as invitro.
* Coumarin and phenindione are used for in-vivo while EDTA is for in-vitro use only.

Your Answer. b
Correct Answer. b

(10). Alcohol intake during pregnancy causes all, except ?

a. Brachycephaly

b. Microcephaly

c. Hyperkinetic disorder

d. Congenital malformation

Solution. Ans-10: (a) Brachycephaly


Ref: Read the text below
Sol:
Fetal alcohol syndrome:
* The diagnosis of FAS typically is based on the observance of a triad of abnormalities in the newborn, including (1) a cluster of
craniofacial abnormalities, (2) CNS dysfunction, and (3) pre- and/or postnatal stunting of growth.
* Facial features (short palpebral fissures, hypoplastic philtrum, and flattened maxilla), and Major organ-system malformations
* CNS anomalies (microcephaly, behavioral abnormalities, and mental retardation),
* Children exposed prenatally to alcohol most frequently present with attentional deficits and hyperactivity, even in the absence of
intellectual deficits or craniofacial abnormalities

Your Answer. a
Correct Answer. a

Copyright 2014 Delhi Academy of Medical Sciences, All Rights Reserved. 5/84
(11). Which teratogen causes deafness?

a. Isotretnoin

b. Chloroquine

c. Alcohol

d. Warfarin

Solution. Ans-11: (d) Warfarin


Ref: Read the text below
Sol:
Warfarin
* Contradi Syndrome : nasal hypoplasia and stippled epiphyseal calcifications
* Limb hypoplasia , hearing loss and ophthalmic anomalies
* dorsal midline dysplasia (agenesis of corpus callosum and DandyWalker malformations) or ventral midline dysplasia (optic atrophy)
Isotretinoin
* CNS defects - hydrocephalus, optic-nerve blindness, retinal defects, microphthalmia, posterior fossa defects, and cortical and cerebellar
defects
* Craniofacial defects (microtia or anotia, low-set ears, hypertelorism, depressed nasal bridge, microcephaly, micrognathia, and agenesis
or stenosis of external ear canals)
* CVS (transposition of great vessels, tetralogy of Fallot, and ventricular or atrial septal defects);
* Thymic defects (ectopia and hypoplasia or aplasia); and
* Miscellaneous defects (limb reduction, decreased muscle tone, spontaneous abortion, and behavioral abnormalities)

Your Answer. d
Correct Answer. d

(12). All of the following drugs are correctly matched with their metabolism except

a. Calcium channel blockers CYP3A4

b. Carvedilol CYP2D6

c. Digoxin P glycoprotein

d. Simvastatin Glucuronide conjugation

Solution. Ans-12: (d) Simvastatin Glucuronide conjugation


Ref: Read the text below
Sol:

* Calcium channel blockers are metabolized by CYP3A4 expressed in the small bowel.
* Carvedilol CYP2D6
* Extensively metabolized by oxidation by CYP2D6 and CYP2C9
* Inhibition of P-glycoprotein may play a role in cases of digoxin toxicity.
* Statins are metabolized by CYP3A4

Your Answer. d
Correct Answer. d

Copyright 2014 Delhi Academy of Medical Sciences, All Rights Reserved. 6/84
(13). Highly plasma protein bound opioid among the following is :

a. Pethidine

b. Fentany1

c. Sufentany1

d. Meperidine

Solution. Ans-13: (c) Sufentany1


Ref.: Miller - 4th ed.
Sol :
Opioids :
* Drugs with high lipid solubility, high unionised fraction or low protein binding in the plasma, demonstrate large volumes of distribution.
* Most opioids are extensively distributed in the body and their volumes of distribution exceed total body water.
* Small intravenous dose of short acting opioid (like alfentanil, sufentanil or fentany1) produce short duration of action because plasma
(and brain) concentration remain above the threshold for therapeutic action for only a brief period as the drug rapidly redistributes from
the CNS to other tissues.

Your Answer. c
Correct Answer. c

(14). Substance which acts by decreasing cAMP?

a. Nor epinephrine via alpha 2 receptor

b. ADH via V2 receptor

c. Nor epinephrine via beta 2 receptor

d. Acetyl choline via M1 receptor

Solution. Ans-14: (a) Nor epinephrine via alpha 2 receptor


Ref.: Katzung 11th ed. Pg. 129
Sol :
* Norepinephrine acts through alpha 2 receptor which is a GPCR.
* The G protein involved here is Gi and it acts by inhibition of cAMP.
* Other receptors where cAMP is inhibited are Muscarinic2, Alpha2, Dopaminergic2 .

Your Answer. c
Correct Answer. a

(15). A 63-year old male with a h/o smoking for the past 20 years presents with an acute onset of left-sided chest pain radiating down the left
arm with profuse sweating. An immediate ECG was performed in the emergency room which revealed a ST-segment elevation in leads
V4-V6.For the purpose of pharmacological thrombolysis, 30 mg of streptokinase was infused intravenously. The immediate plasma
concentration of streptokinase was measured to be 0.6 mg/dL. What is the volume of distribution of streptokinase in this patient?

a. 50 L

b. 5 L

c. 18 L

d. 180 L

Solution. Ans-15: (b) 5 L


Ref.: Read the text below
Sol :
5 L.
Vd = Dose administered / Plasma concentration
= 30 mg/0.6 mg/dL
= 50 dL = 5L

Your Answer. a
Correct Answer. b

Copyright 2014 Delhi Academy of Medical Sciences, All Rights Reserved. 7/84
(16). All of the following are non selective -blockers with additional actions except

a. Carvedilol

b. Betoxalol

c. Oxprenolol

d. Labetolol

Solution. Ans-16: (b) Betoxalol


Ref: Read the text below
Sol:
Nonselective agents
* Alprenolol
* Bucindolol
* Carteolol
* Carvedilol (has additional -blocking activity)
* Labetalol (has additional -blocking activity)
* Nadolol
* Oxprenolol (has intrinsic sympathomimetic activity)
* Penbutolol (has intrinsic sympathomimetic activity)
* Pindolol (has intrinsic sympathomimetic activity)
* Propranolol
* Sotalol
* Timolol
* Eucommia bark (herb)

Your Answer. b
Correct Answer. b

(17). Anti-Platelet therapy is usually given for patients with stroke, MI and peripheral vascular disease. PPIs are frequently administered along
with these drugs to prevent the risk of increased gastrointestinal erosions and bleeding. The interaction between clopidogrel and PPI has
recently been given much attention due to its clinical significance. The metabolizing enzyme common to these two drugs is

a. CYP2C10

b. CYPA2

c. CYP2C19

d. CYP2C20

Solution. Ans-17: (c) CYP2C19


Ref: Read the text below
Sol:
* Clopidogrel is converted in the body to its active form by an enzyme called CYP2C19.
* The studies confirmed that omeprazole can reduce the levels of the active form of clopidogrel in the blood and reduce its antiplatelet
effects, therefore supporting the conclusion that there is an interaction between clopidogrel and omeprazole and esomeprazole.
* There are no solid grounds to extend the warning to other PPIs.

Your Answer. c
Correct Answer. c

Copyright 2014 Delhi Academy of Medical Sciences, All Rights Reserved. 8/84
(18). Anti-Adrenergic drug which crosses the blood brain barrier minimally is -

a. Propranolol

b. Atenolol

c. Oxprenolol

d. Alprenolol

Solution. Ans-18: (b) Atenolol


Ref: Read the text below
Sol:
* Atenolol is a selective 1 receptor antagonist, a drug belonging to the group of beta blockers (sometimes written -blockers), a class of
drugs used primarily in cardiovascular diseases.
* Atenolol was developed as a replacement for propranolol in the treatment of hypertension. The chemical works by slowing down the
heart and reducing its workload.
* Unlike propranolol, atenolol does not pass through the bloodbrain barrier thus avoiding various central nervous system side effects.

Your Answer. a
Correct Answer. b

(19). Side effect of Beta 2 agonist are all except

a. Hyperglycemia

b. Pulmonary odema

c. Tachycardia

d. Tremors

Solution. Ans-19: (a) Hyperglycemia


Ref: Read the text below
Sol:
Findings indicate that Beta-2 stimulants, especially in parenteral administration such as inhalation or injection, can induce adverse
effects:
* Tachycardia secondary to peripheral vasodilation and cardiac stimulation; tachycardia can be accompanied by palpitations.
* Tremor, sweats, agitation.
* More severe effects, pulmonary edema, myocardial ischemia, cardiac arrhythmia, are exceptional.
Asthma aggravation has been observed in patients using large dose of beta-2 stimulants, but it is not known if it results from
spontaneous course of the disease or adverse effect of the drugs. The excipients, in particular sulphite, could contribute to the adverse
effects. The possible loss of the bronchodilator activity of beta-2-mimetic could be attenuated by glucocorticoid intake.

Your Answer. a
Correct Answer. a

Copyright 2014 Delhi Academy of Medical Sciences, All Rights Reserved. 9/84
(20). Mobius syndrome in fetus can caused by antenatal intake of which of the following?

a. Mifepristone

b. Misoprostol

c. Dinoprostone

d. Methotrexate

Solution. Ans-20: (b) Misoprostol


Ref: Read the text below
Sol:
The use of drugs and a traumatic pregnancy may also be linked to the development of Mobius syndrome. The use of the drugs
misoprostol or thalidomide by women during pregnancy has been linked to the development of Mbius syndrome in some cases.
Misoprostol is used to induce abortions
Mbius syndrome (also spelled Moebius) is an extremely rare congenital neurological disorder which is characterized by facial paralysis
and the inability to move the eyes from side to side. Most people with Mbius syndrome are born with complete facial paralysis and
cannot close their eyes or form facial expressions. Limb and chest wall abnormalities sometimes occur with the syndrome.

Your Answer. c
Correct Answer. b

(21). Which of the following antibiotic is a glycopeptide?

a. Clindamycin

b. Vancomycin

c. Azithromycin

d. Linezolid

Solution. Ans-21: (b) Vancomycin


Ref.: Goodman Gilmans - 1539
Sol :
* Vancomycin is atricyclic glycopeptide antibiotic produced by streptococcus orientalis.
* Teicoplanin, a glycopeptide antibiotic produced by Actinoplanes teichomyetius.
* Lipoglycopeptide derivatives telavancin, oritavancin and dalbavancin.
* Rapid intravenous infusion of vancomycin may cause erythematous or urticarial reactions, flushing, tachycardia, and hypotension. The
extreme flushing that can occur is sometimes called red-neck or red-man syndrome.
* Daptomycin is a lipopeptide which is inactivated by pulmonary surfactants.

Your Answer. b
Correct Answer. b

(22). Which should not be given in liver failure?

a. Acetozolamide

b. Furosemide

c. Epelerone

d. Spironolactone

Solution. Ans-22: (a) Acetozolamide


Ref.: Katzung - 256
Sol :
* Carbonic anhydrase inhibitor induced alkalinization of the urine decreases urinary excretion of NH4+ (by converting it to rapidly
reabsorbed NH3) and may contribute to the development of hyperammonemia and hepatic encephalopathy in patients with cirrhosis.
* Another carbonic anhydrase inhibitor, sulthiame, is an anticonvulsant under clinical trials in the USA.

Your Answer. b
Correct Answer. a

Copyright 2014 Delhi Academy of Medical Sciences, All Rights Reserved. 10/84
(23). Anticraving drug used in alcohol dependence?

a. Naltrexone

b. Methadone

c. Vareniciline

d. Rimonabant

Solution. Ans-23: (a) Naltrexone


Ref.: Katzung 11th p. 566
Sol :
* The 1 opioid receptor antagonist and partial agonist naltrexone is FDA-approved for reducing craving in opioid and alcohol addiction.
* Its effect is modest and may involve a modulation of endogenous opioid systems.
* Clinical trials are currently being conducted with a number of drugs, including the high-affinity GABA B-receptor agonist BACLOFEN,
and initial results have shown a significant reduction of craving.
* This effect may be mediated by the inhibition of the dopamine neurons of the VTA, which is possible at baclofen concentrations
obtained by oral administration because of its very high affinity for the GABA-B receptor.

Your Answer. a
Correct Answer. a

(24). An antidepressant found to be associated with tardive dyskinesia and neuroleptic malignant syndrome is -

a. Fluxetine

b. Amineptin

c. Amoxapine

d. Trazodone

Solution. Ans-24: (c) Amoxapine


Ref: Read the text below
Sol:
* Amoxapine (Amokisan, Asendin, Asendis, Defanyl, Demolox, Moxadil) is a tetracyclic antidepressant of the dibenzoxazepine family,
though it is often classified as a secondary amine tricyclic antidepressant. It is the N-demethylated metabolite of Loxapine.
* Common side effects of amoxapine include hypotension, drowsiness, dry mouth, constipation, blurred vision, fatigue, and vertigo.
* Additionally, due to the drug's and its metabolite 7-hyroxyamoxapine's potent blockade of dopamine receptors, it can cause neuroleptic
malignant syndrome as well as acute extrapyramidal symptoms and tardive dyskinesia.
* Cardiovascular and anticholinergic side effects are much reduced compared to other tri- and tetracyclic antidepressants

Your Answer. d
Correct Answer. c

(25). Which of the following drugs causes Retrograde ejaculation as a side effect ?

a. Prazosin

b. Terazosin

c. Alfuzosin

d. Tamsulosin

Solution. Ans-25: (d) Tamsulosin


Ref: Read the text below
Sol:
* Tamsulosin has also affected sexual function in men. It can cause males to experience retrograde ejaculation.
* In males, retrograde ejaculation occurs when the fluid to be ejaculated, which would normally exit the body via the urethra, is
redirected to the urinary bladder.
* Normally, the bladder sphincter contracts and the ejaculate goes to the urethra, the area of least pressure. In retrograde ejaculation,
this sphincter does not function properly.

Your Answer. d
Correct Answer. d

Copyright 2014 Delhi Academy of Medical Sciences, All Rights Reserved. 11/84
(26). All are side effects of valproic acid except ?

a. Polycystic ovaries

b. Fulminant hepatitis

c. Tremors

d. Lymadenopathy

Solution. Ans-26: (d) Lymadenopathy


Ref: Read the text below
Sol:
Adverse effects are dosage-related.
* The foremost and most severe concern for anyone taking valproic acid is its potential for sudden and severe, possibly fatal, fulminating
impairments in liver and impairments of hematopoietic or pancreatic function, especially in those just starting the medication.
* Valproate is also cautioned against in many patients because it can cause weight gain.
* Common side effects are dyspepsia or weight gain. Less common are fatigue, peripheral edema, acne, feelings of feeling cold or chills,
blurred vision, burning of the eyes, dizziness, drowsiness, hair loss, headaches, nausea, sedation, and tremors. Valproic acid also causes
hyperammonemia, an increase of ammonia levels in the blood, which can lead to vomiting and sluggishness, and ultimately to mental
changes and brain damage. Valproate levels within the normal range are capable of causing hyperammonemia and ensuing
encephalopathy.

Your Answer. d
Correct Answer. d

(27). Which organ is not affected in lithium toxicity?

a. Brain

b. Liver

c. Heart

d. Kidney

Solution. Ans-27: (b) Liver


Ref: Read the text below
Sol:
* Lithium toxicity may occur in persons taking excessive amounts either accidentally or intentionally on an acute basis or in patients who
accumulate high levels during ongoing chronic therapy.
* The manifestations include nausea, emesis, diarrhea, asthenia, ataxia, confusion, lethargy, polyuria, seizures and coma. Other toxic
effects of lithium include coarse tremor, muscle twitching, convulsions and renal failure.
* People who survive a poisoning episode may develop persistent neurotoxicity. Several authors have described a "Syndrome of
Irreversible Lithium-Effected Neurotoxicity" (SILENT), associated with episodes of acute lithium toxicity or long-term treatment within
the appropriate dosage range. Symptoms are said to include cerebellar dysfunction

Your Answer. a
Correct Answer. b

Copyright 2014 Delhi Academy of Medical Sciences, All Rights Reserved. 12/84
(28). Lithium clearance is decreased by

a. Hyperkalemia

b. Hypokalemia

c. Hyponatremia

d. Hypernatremia

Solution. Ans-28: (c) Hyponatremia


Ref: Read the text below
Sol:
* Lithium is minimally protein bound (< 10%) and has an apparent volume of distribution of 0.6-1 L/kg. The therapeutic dose is 300-2700
mg/d with desired serum levels of 0.6-1.2 mEq/L.
* Lithium clearance is predominantly through the kidneys. Because it is minimally protein bound, lithium is freely filtered at a rate that is
dependent upon the glomerular filtration rate (GFR). Consequently, dosing must be adjusted based on renal function. Individuals with
chronic renal insufficiency must be closely monitored if placed on lithium therapy.
* Most filtered lithium is reabsorbed in the proximal tubule; thus, drugs known to inhibit proximal tubular reabsorption, such as carbonic
anhydrase inhibitors and aminophylline, may increase excretion. Diuretics acting distally to the proximal tubule, such as thiazides and
spironolactone, do not directly affect the fractional excretion of lithium (although they may affect serum lithium levels indirectly through
their effects on volume status). Reabsorption of lithium is increased and toxicity is more likely in patients who are hyponatremic or
volume depleted, both of which are possible consequences of diuretic therapy.

Your Answer. a
Correct Answer. c

(29). All are true except :

a. Famciclovir is the prodrug of penciclovir

b. Penciclovir does not cause chain termination

c. Penciclovir triphosphate has lower affinity for the viral DNA polymerase than acyclovir triphosphate

d. Incidence of mammary adenocarcinoma increased in female rats.

Solution. Ans-29: (a) Famciclovir is the prodrug of penciclovir


Ref.: Katzung 11th ed.p. 848
Sol :
* Famciclovir is the diacetyl ester prodrug of 6-deoxypenciclovir, an acyclie guanosine analog.
* After oral administration, famciclovir is rapidly deacetylated and oxidized by first-pass metabolism to penciclovir.
* Unlike acyclovir penciclovir does not cause chain termination.
* Penciclovir triphosphate has lower affinity for the viral DNA polymerase thanacyclovir triphosphate, but it achieves higher intracellular
concentrations.
* The most commonly encountered clinical mutants of HSV are thymidine kinase-deficient; these are cross-resistant to acyclovir and
famciclovir.
* As with acyclovir, testicular toxicity has been demonstrated in animals receiving repeated doses.
* The incidence of mammary adenocarcinoma was increased in female rats receiving famciclovir for 2 years.

Your Answer. a
Correct Answer. a

Copyright 2014 Delhi Academy of Medical Sciences, All Rights Reserved. 13/84
(30). Diarrhea is a side effect of :

a. Amitryptiline

b. Metformin

c. Atropine

d. Codeine

Solution. Ans-30: (b) Metformin


Ref.: Katzung 11th ed.p. 742
Sol :
* The most common toxic effects of metformin are gastrointestinal (anorexia, nausea, vomiting, abdominal discomfort, and diarrhea),
which occur in up to 20% of patients.
* Metformin may have to be discontinued in 3-5% of patients because of persisten diarrhea.
* In the absence of hypoxia or renal or hepatic insufficiency, lactic acidosis is less common with metformin thereapy than with
phenformin therapy.

Your Answer. a
Correct Answer. b

(31). All are true about Buspirone except :

a. Not a Benzodiapezine

b. Is a anti anxiety drug

c. Used as sedative hypnotic

d. Used as anti depressant

Solution. Ans-31: (c) Used as sedative hypnotic


Ref.: Katzung 11th ed.p. 374
Sol :
* Buspirone is a non benzodiazepine arylpiperazine used as anxiolytic and antidepressants.
* Gepirone and ipsapirone are other drugs in this group.
* It does not interact with GABA-A receptors and/or display the sedative and anticonvulsant properties of benzodiazepines.
* It may exert its anxiolytic effects by acting as a partial agonist at brain 5-HT1-A receptors, but it also has affinity for brain dopamine D2
receptors.
* The drug is used in generalized anxiety states but is less effective in panic disorders.
* Nonspecific chest pain, tachycardia, palpitations, dizziness, nervousness, tinnitius, gastrointestinal distress, and paresthesias and a
dose-dependent pupillary constriction may occur.

Your Answer. c
Correct Answer. c

(32). A patient of peptic ulcer was prescribed ranitidine and sucralfate in the morning hours. Why is this combination incorrect?

a. Ranitidine combines with sucralfate and prevents its action

b. Combination of these two drugs produces serious side effects like agranulocytosis

c. Ranitidine decreases the gastric pH so sucralfate is not able to act.

d. Sucralfate inhibits absorption of ranitidine

Solution. Ans-32: (d) Sucralfate inhibits absorption of ranitidine


Ref.: Read the text below
Sol :
* Ranitidine can increase the gastric pH and prevent polymerization (occurs at a pH < 4) of sucralfate.
* Sucralfate decreases the absorption of many drugs including ranitidine.

Your Answer. a
Correct Answer. d

Copyright 2014 Delhi Academy of Medical Sciences, All Rights Reserved. 14/84
(33). Which of the following fluoroquinolones does not require dose adjustment in a patient with creatinine clearance of < 50 mg/minute?

a. Ciprofloxacin

b. Trovafloxacin

c. Lomefloxacin

d. Sparfloxacin

Solution. Ans-33: (b) Trovafloxacin


Ref.: Read the text below
Sol :
* Trovafloxacin is a broad spectrum antibiotic that inhibits the uncoiling of supercoiled DNA in various bacteria by blocking the activity of
DNA gyrase and topoisomerase IV.
* It was withdrawn from the market due to the risk of hepatotoxicity.
* It had better gram-positive bacterial coverage and less gram-negative coverage than the previous fluoroquinolones.

Your Answer. a
Correct Answer. b

(34). Which of the following is the best indication for propofol as an intravenous indication agent?

a. Neurosurgery

b. Day care surgery

c. Patients with coronary artery disease

d. In neonates

Solution. Ans-34: (b) Day care surgery


Ref.: Read the text below
Sol :
* Propofol is contraindicated in children.

Your Answer. b
Correct Answer. b

(35). Which of the following recent drugs for Hepatitis C has been on hold due to reports of severe pancreatitis?

a. Alisporivir

b. Daclatasvir

c. Ladipasvir

d. Danoprevir

Solution. Ans-35: (a) Alisporivir


Ref.: Read the text below
Sol :
* Alisporivir is a cyclosporin derivative. It inhibits cyclophilin A.
* Daclatasvir and Ladipasvir are NS5a inhibitors. They inhibit formation of membrane-bound replication complex
* Danoprevir is a NS3-NS4a protease inhibitor. It inhibits proteolysis of HCV protein precursors.

Your Answer. b
Correct Answer. a

Copyright 2014 Delhi Academy of Medical Sciences, All Rights Reserved. 15/84
(36). Which of the following is recently approved by U.S. F.D.A. for the treatment of psoriatic arthritis?

a. Tasimelteon

b. Apremilast

c. Ibrutinib

d. Droxidopa

Solution. Ans-36: (b) Apremilast


Ref.: Read the text below
Sol :
Tasimelteon is a melatonin (MLT-1 and MLT-2) agonist approved for the treatment of non-24-hour sleepwake disorder in totally blind
people on 31st Jan 2014.
* Apremilast is a PDE-4 inhibitor approved for psoriatic arthritis on 21st Mar 2014.
* Ibrutinib is a Brutons tyrosine kinase inhibitor approved for mantle cell lymphoma and CLL on 12th Feb 2014.
* Droxidopa is a prodrug of NA/Adr approved for neurogenic orthostatic hypotension, dialysis-induced hypotension and hypotension
associated with fibromyalgia and chronic fatigue syndrome on 18th Feb 2014.

Your Answer. c
Correct Answer. b

(37). The anti-TB capsule RISORINE contains all of the following except

a. Isoniazid

b. Rifampicin

c. Pyrazinamide

d. Piperine

Solution. Ans-37: (c) Pyrazinamide


Ref.: Read the text below
Sol :
Pyrazinamide
Contents of Risorine:
* Isoniazid (300 mg)
* Rifampicin (200 mg)
* Piperine (10 mg) (Bioenhancer. Inhibits rifampicin metabolism; hence permits reduction in the dose of rifampicn)

Your Answer. d
Correct Answer. c

Copyright 2014 Delhi Academy of Medical Sciences, All Rights Reserved. 16/84
(38). Low dose inhaled corticosteroids (ICS) + Long-acting inhaled 2 agonist therapy is the preferred treatment for which type of bronchial
asthma?

a. Mild intermittent asthma

b. Mild persistent asthma

c. Moderate persistent asthma

d. Severe persistent asthma

Solution. Ans-38: (c) Moderate persistent asthma


Ref.: Read the text below
Sol :
Moderate persistent asthma

Your Answer. b
Correct Answer. c

(39). Which of the following drugs is a dual PPAR agonist recently approved for diabetic dyslipidemia?

a. Muraglitazar

b. Tesaglitazar

c. Aleglitazar

d. Saroglitazar

Solution. Ans-39: (d) Saroglitazar


Ref.: Read the text below
Sol :
Saroglitazar. It was approved on 5th June 2013.
* Muraglitazar was withdrawn after Phase 3 due to increased risk of CV events
* Tesaglitazar was withdrawn after Phase 3 due to high nephrotoxicity
* Aleglitazar was abandoned in June 2013 due to high risk of bone fractures, heart failure and GI bleeding
All of these are dual PPAR ( and ) inhibitors.

Your Answer. d
Correct Answer. d

Copyright 2014 Delhi Academy of Medical Sciences, All Rights Reserved. 17/84
(40). Theophylline levels are increased by which of the following drugs?

a. Ciprofloxacin

b. OCPs

c. Erythromycin

d. All of the above

Solution. Ans-40: (d) All of the above


Ref.: Read the text below
Sol :

Your Answer. d
Correct Answer. d

(41). A patient with intractable itching would best respond to which of the following?

a. Chlorpromazine

b. Pimozide

c. Haloperidol

d. Risperidone

Solution. Ans-41: (a) Chlorpromazine


Ref: Read the text below
Sol :
* Agents with H1 receptor blocking actions are effective in reducing itching. H1 receptor blockade is typical of phenothiazines with short
side chains.

Your Answer. Not Attempted


Correct Answer. a

Copyright 2014 Delhi Academy of Medical Sciences, All Rights Reserved. 18/84
(42). Zero order kinetic is shown by all except

a. High dose salicylates

b. Phenytoin

c. Ethanol

d. Methotrexate

Solution. Ans-42: (d) Methotrexate


Ref: KDTs- 28
Sol:
* Zero order kinetics Rate of elimination remains constant irrespective of drug concentration e.g. ethyl alcohol.
* First Order Kinetics : Elimination of the Drug is directly proportional to its plasma Concerntration. its dependent on its Half life. First
order implies that No matter How much concerntration of the Drug u give it will be eliminated 50% by its First Half- life. Tht means its
time dependent. Like if the drug has a half life of 30 mins and u give 2 gms then after 30mins only 1 gm will be left in the plasma. The
50% of active form of the drug that will disappear from the Plasma in a certain amount of time is termed as Half Life.
* Zero Order Elimination: This form of elimination is refered to as elimination which is independent of time or concerntration. it will take
a linear approach on a graph
* Drugs whose kinetics changes from 1st order to zero order over the therapeutic range are phenytoin, tolbutamide, theophylline,
warfarin and salicylate (aspirin).

Your Answer. d
Correct Answer. d

(43). The process by which the amount of a drug in the body decreases after administration but before entering the systemic circulation is
called

a. Excretion

b. First pass effect

c. First order elimination

d. Metabolism

Solution. Ans-43: (b) First pass effect


Ref: Katzung - 41
Sol:
* The first-pass effect (also known as first-pass metabolism or presystemic metabolism) is a phenomenon of drug metabolism whereby the
concentration of a drug is greatly reduced before it reaches the systemic circulation.
* It is the fraction of lost drug during the process of absorption which is generally related to the liver and gut wall.
* Notable drugs that experience a significant first-pass effect are imipramine, morphine, propranolol, buprenorphine, diazepam,
midazolam, demerol, cimetidine, and lidocaine.
* Reduction in the amount of drug before it enters the systemic circulation is called first pass metabolism (also known as first pass effect)
whereas if the amount of drug decreases after entry into the systemic circulation, it is called elimination.Latter includes excretion and
metabolism.

Your Answer. b
Correct Answer. b

Copyright 2014 Delhi Academy of Medical Sciences, All Rights Reserved. 19/84
(44). Physiological antagonism is found in

a. Isoprenaline and salbutamol

b. Isoprenaline and adrenaline

c. Isoprenaline and propanolol

d. Adrenaline and histamine

Solution. Ans-44 (d) Adrenaline and histamine


Ref: Read the text below.
Sol:
* There are several physiological antagonists that have antihistaminergic action. For instance, adrenaline raises arterial pressure
through vasoconstriction mediated by -adrenergic receptor activation, in contrast to the histamine effect of lowering arterial pressure.
However, only such substances that bind and block the histamine receptor are true antihistamines.
* Isoprenaline and salbutamol or adrenaline are not antagonists at all.
* Isoprenaline and propranolol are pharmacological antagonists because they are acting on same receptors.

Your Answer. d
Correct Answer. d

(45). Epinephrine causes hyperglycemia by

a. Increased glucagon

b. Decreased insulin secretion

c. Increased glucocorticoids

d. Increased thyroxine

Solution. Ans-45: (a) Increased glucagon


Ref: KDTs - 124
Sol:
* Epinephrine from the adrenal medulla acts via beta-adrenergic receptors, whereas norepinpherine is released from nerve endings and
acts on alpha2-adrenergic receptors.
* Norepinephrine and epinephrine have somewhat opposing effects on insulin release (norepinephrine inhibits, epinephrine stimulates),
but the net effect of both is increased blood glucose.
* This occurs via stimulation of glycogenolysis and release of glucose from hepatocytes (epinephrine), and indirectly through inhibition of
insulin release (norepinephrine), and release of growth hormone (epinephrine) and ACTH (which increases cortisol).

Your Answer. a
Correct Answer. a

(46). A 26-year-old female with reactive depression complains of missing her period and having milk discharge from her breasts. She has no
signs of pregnancy, including a negative pregnancy test. Which of the following might have caused these findings?

a. Clomipramine

b. Amoxapine

c. Fluoxetine

d. Mirtazapine

Solution. Ans-46: (b) Amoxapine


Ref: Read the text below
Sol :
* Amoxapine is a heterocyclic antidepressant that has effects on norepinephrine and serotonin uptake. It is useful in psychotic patients
who are depressed.
* The dopaminergic antagonism caused by amoxapine may lead to the amenorrheagalactorrhea syndrome.

Your Answer. c
Correct Answer. b

Copyright 2014 Delhi Academy of Medical Sciences, All Rights Reserved. 20/84
(47). All of the following drugs are used topically in the treatment of chronic wide-angle glaucoma. Which of these agents reduces intraocular
pressure by decreasing the formation of the aqueous humor?

a. Timolol

b. Echothiophate

c. Pilocarpine

d. Isofluorphate

Solution. Ans-47: (a) Timolol


Ref: Read the text below
Sol :
* When applied topically to the eye, both the direct-acting cholinomimetic agents (e.g., pilo- carpine) and those cholinomimetic drugs that
act by inhibition of AChE (e.g., echothiophate, isofluorphate, and physostigmine) cause miosis by contracting the sphincter muscle of the
iris and reducing ocular pressure by contracting the ciliary muscle.
* In patients with glaucoma, this latter effect permits greater drainage of the aqueous humor through the trabecular meshwork in the
canal of Schlemm and a reduction in resistance to outflow of the aqueous humor. Certain -adrenergic blocking agents (e.g., timolol and
levobunolol) applied to the eye are also very useful in treating chronic wide-angle glaucoma.
* These drugs appear to act by decreasing the secretion (or formation) of the aqueous humor by antagonizing the effect of circulating
catecholamines on -adrenergic receptors in the ciliary epithelium.

Your Answer. c
Correct Answer. a

(48). Neostigmine will effectively antagonize skeletal muscle relaxation produced by

a. Pancuronium

b. Succinylcholine

c. Diazepam

d. Baclofen

Solution. Ans-48: (a) Pancuronium


Ref: Read the text below
Sol :
* Anticholinesterase agents, such as neostigmine, will delay the catabolism of ACh that is released from parasympathetic autonomic and
somatic nerve terminals.
* At the neuromuscular junction, this results in increased competition for the NM receptors by ACh (the agonist) and the curariform
drugs (the antagonists) such as tubocurarine and pancuronium. In addition, neostigmine has a direct stimulating action on the skeletal
muscle junction, which enhances its ability to antagonize the competitive neuromuscular blockers.
* The activity of succinylcholine at the neuromuscular junction will be exacerbated by neostigmine, because succinylcholine is inactivated
by AChE. The skeletal muscle relaxation that may result from toxic doses of nicotine-blocking NM receptors will be unaffected by
neostigmine.
* Diazepam and baclofen are centrally acting skeletal muscle relaxants whose effects are not altered by the peripheral actions of
neostigmine.

Your Answer. a
Correct Answer. a

Copyright 2014 Delhi Academy of Medical Sciences, All Rights Reserved. 21/84
(49). Epinephrine may be mixed with certain anesthetics, such as procaine, in order to

a. Stimulate local wound repair

b. Promote hemostasis

c. Retard their systemic absorption

d. Facilitate their distribution along nerves and fascial planes

Solution. Ans-49: (c) Retard their systemic absorption


Ref: Read the text below
Sol :
* The addition of a vasoconstrictor, such as epinephrine or phenylephrine, to certain short-acting, local anesthetics is a common practice
in order to prevent the rapid systemic absorption of the local anesthetics, to prolong the local action, and to decrease the potential
systemic reactions.
* Some local anesthetics cause vasodilation, which allows more compound to escape the tissue and enter the blood.
* Procaine is an ester-type local anesthetic with a short duration of action due to rather rapid biotransformation in the plasma by
cholinesterases.
* The duration of action of the drug during infiltration anesthesia is greatly increased by the addition of epinephrine, which reduces the
vasodilation caused by procaine.

Your Answer. c
Correct Answer. c

(50). Both phentolamine and prazosin

a. Are competitive antagonists at 1-adrenergic receptors

b. Have potent direct vasodilator actions on vascular smooth muscle

c. Enhance gastric acid secretion through a histamine-like effect

d. Cause hypotension and bradycardia

Solution. Ans-50: (a) Are competitive antagonists at 1-adrenergic receptors


Ref: Read the text below
Sol :
* Phentolamine is a nonselective -adrenergic receptor blocker (i.e., it has affinity for both 1- and 2-adrenergic receptor sites). It also
has a prominent direct relaxant (musculotropic spasmolytic) effect on arterioles, which results in vasodilation and reflex tachycardia.
* In addition, phentolamine can block the effects of serotonin and will increase hydrochloric acid and pepsin secretion from the stomach.
Phentolamine is used for the short-term control of hypertension in patients with pheochromocytoma (i.e., a type of secondary
hypertension); because of the high incidence of tachycardia associated with the compound, it is not used chronically for the treatment of
essential hypertension.
* Prazosin is a selective 1-adrenergic receptor antagonist that, at therapeutic doses, has little activity at 2-adrenergic receptors and
clinically insignificant direct vasodilating activity. The drug does not cause the other effects attributed to phentolamine. Most important,
it produces less tachycardia than does phentolamine and, therefore, is useful in the treatment of essential hypertension.

Your Answer. a
Correct Answer. a

Copyright 2014 Delhi Academy of Medical Sciences, All Rights Reserved. 22/84
(51). A 10-year-old male displays hyperactivity and is unable to focus on his schoolwork because of an inability to focus on the activity. Which
of the following might prove effective in this patient?

a. Methylphenidate

b. Terbutaline

c. Dobutamine

d. Pancuronium

Solution. Ans-51: (a) Methylphenidate


Ref: Read the text below
Sol :
* Methylphenidate is similar to amphetamine and acts as a CNS stimulant, with more pronounced effects on mental than on motor
activities.
* It is effective in the treatment of narcolepsy and attention-deficit hyperactivity disorders.

Your Answer. a
Correct Answer. a

(52). Nicotine in low doses may cause

a. Decreased tone and motor activity of the small intestine

b. Stimulation of the respiratory rate and depth

c. Miosis

d. Bradycardia

Solution. Ans-52: (b) Stimulation of the respiratory rate and depth


Ref: Read the text below
Sol :
* Nicotine is a depolarizing ganglionic blocking agent; that is, it stimulates nicotinic receptors in low doses and predominantly blocks at
high-dose levels.
* The effect of nicotine on a particular tissue or organ depends on the relative contribution to the function made by each division of the
ANS.
* The effects on the cardiovascular system are complex. Stimulation of the cardiac vagal ganglia causes bradycardia. This is countered by
sympathetic stimulation to the heart (tachycardia), blood vessels (vasoconstriction), and adrenal medulla (catecholamine release:
tachycardia and vasoconstriction). Thus, the net effect of nicotine on the heart is tachycardia, not bradycardia.
* Low doses of nicotine augment respiration by excitation of the chemoreceptors of the carotid body and aortic arch. Higher doses also
stimulate the medullary respiratory center and increase respiration through CNS activity.
* Large amounts of nicotine cause respiratory failure from medullary paralysis and blockade of the skeletal muscles of respiration.

Your Answer. b
Correct Answer. b

Copyright 2014 Delhi Academy of Medical Sciences, All Rights Reserved. 23/84
(53). A naturally occurring substance that is useful in treating Pagets disease of the bone is

a. Etidronate

b. Cortisol

c. Calcitonin

d. Parathyroid hormone (PTH)

Solution. Ans-53: (c) Calcitonin


Ref: Read the text below
Sol :
* Calcitonin is useful in the therapy of Pagets disease of bone (osteitis deformans). Calcitonin therapy reduces urinary hydroxyproline
excretion and serum alkaline phosphatase activity and provides some symptomatic relief.
* Presumably, these effects result from the ability of calcitonin to inhibit bone resorption. Side effects of long-term therapy with this
hormone can include nausea, edema of the hands, and urticaria. The appearance of neutralizing antibodies may explain the development
of resistance to treatment.
* Etidronate is a synthetic drug that is useful in Pagets disease. The compound is orally effective and lacks the antigenicity associated
with calcitonin.

Your Answer. a
Correct Answer. c

(54). A patient becomes markedly tetanic following a recent thyroidectomy. This symptom can be rapidly reversed by the administration of

a. Vitamin D

b. Calcitonin

c. Plicamycin (mithramycin)

d. Calcium gluconate (CaG)

Solution. Ans-54: (d) Calcium gluconate (CaG)


Ref: Read the text below
Sol :
* Administration of intravenous CaG would immediately correct the tetany that might occur in a patient in whom a thyroidectomy was
recently performed.
* Parathyroid hormone would act more slowly but could be given for its future stabilizing effect.
* Long-term control of a patient after a thyroidectomy can be obtained with vitamin D and dietary therapy.
* Calcitonin is a hypocalcemic antagonist of parathyroid hormone. Plicamycin (mithramycin) is used to treat Pagets disease and
hypercalcemia. The dose employed is about one-tenth the amount used for plicamycins cytotoxic action.

Your Answer. d
Correct Answer. d

Copyright 2014 Delhi Academy of Medical Sciences, All Rights Reserved. 24/84
(55). A 55-year-old postmenopausal female develops weakness, polyuria, and polydipsia. Nephrocalcinosis is detected by a computed
tomography (CT) scan. Her serum creatinine is elevated. Which of the following agents may have caused these adverse effects?

a. Estrogens

b. Prednisone

c. Etidronate

d. Vitamin D

Solution. Ans-55: (d) Vitamin D


Ref: Read the text below
Sol :
* Enthusiastic overmedication with vitamin D may lead to a toxic syndrome called hypervitaminosis D.
* The initial symptoms can include weakness, nausea, weight loss, anemia, and mild acidosis.
* As the excessive doses are continued, signs of nephrotoxicity are manifested, such as polyuria, polydipsia, azotemia, and eventually
nephrocalcinosis. In adults, osteoporosis can occur. Also, there is CNS impairment, which can result in mental retardation and
convulsions.

Your Answer. c
Correct Answer. d

(56). If a drug is given repeatedly at approximately the same time as its half life, after how many dosages will the drug achieve a steady state
in plasma?

a. 2-3

b. 4-5

c. 8-9

d. 6-8

Solution. Ans-56: (b) 4-5


Ref.: Read the text below
Sol :
* Steady state is reached in 4-5 half lives unless dose interval is very much longer than t.
* Given during late pregnancy, corticosteroids accelerate lung maturation in the fetus and prevent respiratory distress syndrome at birth.
* Such therapy may be undertaken if premature delivery is contemplated.
* Given during early pregnancy, corticosteroids cause fetal abnormalities such as Cleft palate, harelip, cardiac septal defects,
neurological and behavioral disturbances are likely if corticosteroids are given during pregnancy, especially 2nd trimester.

Your Answer. b
Correct Answer. b

(57). Famotidine and cimetidine are different in their action of :

a. Oral absorption

b. Duration of action

c. Mechanism of action

d. Antiandrogenic action

Solution. Ans-57: (d) Antiandrogenic action


Ref.: Read the text below
Sol :
* Cimetidine (but not other H2 blockers) has anti androgenic action (displaces dihydrotestosterone from its cytoplasmic receptor),
increases plasma prolacin and inhibits degradation of estradiol by liver.
* High doses given for long periods have produced gynaecomastia, loss of libido, impotence and temporary decrease in sperm count.

Your Answer. d
Correct Answer. d

Copyright 2014 Delhi Academy of Medical Sciences, All Rights Reserved. 25/84
(58). Regarding meperidine all are true except :

a. Drug dependence

b. Short half life than morphine

c. More spasmogenic than morphine

d. Action is antagonize by Naloxone

Solution. Ans-58: (c) More spasmogenic than morphine


Ref.: Read the text below
Sol :
* Pethidine was synthesized as an atropine substitute in 1939, and has some actions like it. Though chemically unrelated to morphine, it
interacts with opioid receptors and its actions are blocked by naloxone.
Important differences in comparison to morphine are :
* Dose to dose 1/8 - 1/10 in analgesic potency : However, analgesic efficacy approaches near to morphine and is more than codeine.
* After i.m. injection the onset of action is more rapid but duration is shorter (3-4 hours).
* It does not effectively suppress cough.
* Spasmodic action on smooth muscles is less marked V Miosis, constipation and urinary retention are less prominent.
* Pethidine is believed to induce less biliary spasm than morphine; traditionally preferred in cholecystitis colic. However, there is no
objective evidence to support this belief.
* It is equally sedative and euphoriant has similar abuse potential. The degree of respiratory depression seen at equianalgesic does
equivalent to morphine.

Your Answer. c
Correct Answer. c

(59). Glucocorticoids are powerful anti-inflammatory agents. Which of the following is not an anti-inflammatory mechanism of action of
glucocorticoids?

a. Decreased secretion of proteolytic enzymes

b. Reduction in the release of cytokines, such as IL-1 and IL-2

c. Decreased number of circulating neutrophils

d. Impairment of prostaglandin and leukotriene synthesis

Solution. Ans-59: (c) Decreased number of circulating neutrophils


Reference Read the text below
Sol:
* Glucocorticoids actually increase the number of circulating neutrophils as inhibition of margination and migration occurs.
Paradoxically, this is an anti-inflammatory effect as neutrophils had not been reaching the sites where they were needed.
* All of the other choices are anti-inflammatory mechanisms of glucocorticoid action.

Your Answer. c
Correct Answer. c

Copyright 2014 Delhi Academy of Medical Sciences, All Rights Reserved. 26/84
(60). A 60-year-old diabetic male on an oral hypoglycemic agent develops abnormal liver function tests. Which of the following agents can
cause this finding?

a. Glyburide

b. Metformin

c. Troglitazone

d. Acarbose

Solution. Ans-60: (c) Troglitazone


Reference Read the text below
Sol:
* Hepatic dysfunction has occurred particularly with the use of troglitazone, necessitating its removal from the market.
* Patients treated with other thiazolidinediones should be monitored for this possibility.

Your Answer. c
Correct Answer. c

(61). In patients with hepatic coma, decreases in the production and absorption of ammonia from the gastrointestinal (GI) tract will be
beneficial. Which of the following is the antibiotic of choice in this situation?

a. Neomycin

b. Tetracycline

c. Penicillin G

d. Chloramphenicol

Solution. Ans 61: (a) Neomycin


Ref Read the text below
Sol:
* Neomycin, an aminoglycoside, is not significantly absorbed from the GI tract. After oral administration, the intestinal flora is
suppressed or modified and the drug is excreted in the feces.
* This effect of neomycin is used in hepatic coma to decrease the coliform flora, thus decreasing the production of ammonia that causes
the levels of free nitrogen to decrease in the bloodstream.
* Other antimicrobial agents (e.g., tetracycline, penicillin G, chloramphenicol, and cephalothin) do not have the potency of neomycin in
causing this effect.

Your Answer. a
Correct Answer. a

Copyright 2014 Delhi Academy of Medical Sciences, All Rights Reserved. 27/84
(62). True about ACE Inhibitor ?

a. Conversion of angiotensinogen to angiotensin I

b. Enalapril has a longer life than lisinophil

c. Omission of pretreatment with diuretics minimizes the risk of 1st dose hypotension

d. It is not effective till left systolic dysfunction is not manifested

Solution. Ans 62: (c) Omission of pretreatment with diuretics minimizes the risk of 1st dose hypotension
Ref KDT5th ed- Goodman gilman-9th ed ch-30
Sol:
About Option a-
* Ace inhibitors inhibits formation of angiotensin I to angiotensin II.
About option b-
* t1/2 of enalapril is is 1.3 hrs and its metabolite enalaprilat has t1/2 of 11 hrs but lisinopril has t1/2 of 12 hrs.
About option c-
* A steep fall in blood pressure may occur following the first dose of an ACE inhibitor in patients with elevated PRA.
* Special care should be exercised in patients who are saltdepleted, in patients being treated with multiple antihypertensive drugs, and
in patients who have congestive heart failure.
* Such patients should be started on very small doses of ACE inhibitors, or salt intake should be increased and diuretics stopped before
beginning therapy.
About option d-
* Left ventricular systolic dysfunction ranges from a modest, asymptomatic reduction in systolic performance to a severe impairment of
left ventricular systolic function with florid congestive heart failure.
* Unless contraindicated, ACE inhibitors should be given to all patients with impaired left ventricular systolic function whether or not
they have symptoms of overt heart failure.

Your Answer. a
Correct Answer. c

(63). A drug-drug interaction is possible with sildenafil and which of the following agents?

a. Dobutamide

b. Alprostadil

c. Digoxin

d. Nitrates

Solution. Ans-63: (d) Nitrates


Reference Read the text below
Sol:
* Sildenafil is effective in many patients with erectile dysfunction. It increases cGMP by inhibiting phosphodiesterase isoform 5.
* The potentiation of nitrate action by sildenafil is thought to result from both agents increasing the concentration of nitric oxide, which
leads to an increase in cGMP, thereby resulting in hypotension.

Your Answer. d
Correct Answer. d

Copyright 2014 Delhi Academy of Medical Sciences, All Rights Reserved. 28/84
(64). A 22-year-old female who requests a postcoital contraceptive after being raped would best be treated with which of the following?

a. Mifepristone

b. Spironolactone

c. Aminoglutethimide

d. Leuprolide

Solution. Ans-64: (a) Mifepristone Reference - Read the text below


Sol:
* Mifepristone is structurally related to norethindrone. This compound is classified as a progesterone antagonist with weak agonistic
properties.
* A single dose can function as an emergency postcoital contraceptive. It also can induce an abortion by causing contraction of the
myometrium, which leads to detachment of the embryo.
* The drug is used in single or multiple doses followed by the administration of a prostaglandin to cause the abortion.
* Estrogens used alone or in combination with progestins have also proven effective in postcoital contraception.

Your Answer. a
Correct Answer. a

(65). Which of the following agents will increase pulse pressure?

a. Metoprolol

b. Dopamine

c. Isoproterenol

d. Epinephrine

Solution. Ans 65 : (d) Epinephrine


Ref Read the text below
Sol:
* Epinephrine has a positive ionotropic and chronotropic effect on the heart because of its 1-adrenergic activity.
* It also has -adrenergic activity that causes vasoconstriction in the vascular beds. These actions result in a rise in systolic blood
pressure.
* Epinephrine also has 2-adrenergic activity, which causes vasodilation in skeletal muscle.
* Because of this latter effect, total peripheral resistance can fall, resulting in a drop in diastolic pressure, particularly at low doses of
epinephrine.

Your Answer. b
Correct Answer. d

(66). A male patient is brought to the emergency department (ED) following ingestion of an unknown substance. He is found to have an
elevated temperature, hot and flushed skin, dilated pupils, and tachycardia. Of the following, which would most likely cause these
findings?

a. Propranolol

b. Methylphenidate

c. Guanethidine

d. Atropine

Solution. Ans 66 : (d) Atropine


Ref Read the text below
Sol:
* High concentrations of atropine block all parasympathetic function.
* The patient usually presents with an array of symptoms and signs that include dry mouth, dilated pupils, tachycardia,red and hot skin,
and delirium.
* Hyperthermia may occur, particularly in very young children.

Your Answer. a
Correct Answer. d

Copyright 2014 Delhi Academy of Medical Sciences, All Rights Reserved. 29/84
(67). A 65-year-old male has a blood pressure of 170/105 mmHg. Which of the following would be effective in lowering this patients blood
pressure?

a. Methylphenidate

b. Terbutaline

c. Pancuronium

d. Prazosin

Solution. Ans 67 : (d) Prazosin


Ref Read the text below
Sol:
* Prazosin blocks 1-adrenergic receptors in arterioles, thereby decreasing peripheral resistance and leading to a decrease in blood
pressure.
* Orthostatic hypotension can occur, particularly after a first dose.

Your Answer. d
Correct Answer. d

(68). The EKG of a patient who is receiving digitalis in the therapeutic dose range would be likely to show

a. Prolongation of the QT interval

b. Prolongation of the PR interval

c. Symmetric peaking of the T wave

d. Widening of the QRS complex

Solution. Ans-68: (b) Prolongation of the PR interval


Ref: Read the text below.
Sol:
* The usual electrocardiographic pattern of a patient receiving therapeutic doses of digitalis includes an increase in the PR interval,
depression and sagging of the ST segment, and occasional biphasia or inversion of the T wave.
* Symmetrically peaked T waves are associated with hyperkalemia or ischemia in most cases.
* Shortening of the QT interval, rather than prolongation, is characteristic of digitalis treatment.

Your Answer. d
Correct Answer. b

(69). In a hypertensive patient who is taking insulin to treat diabetes,which of the following drugs is to be used with extra caution and advice
to the patient?

a. Hydralazine

b. Prazosin

c. Guanethidine

d. Propranolol

Solution. Ans-69: (d) Propranolol


Ref: Read the text below.
Sol:
* Propranolol, as well as other nonselective beta blockers, tends to slow the rate of recovery in a hypoglycemic attack caused by insulin.
* Beta blockers also mask the symptoms of hypoglycemia and may actually cause hypertension because of the increased plasma
epinephrine in the presence of a vascular beta2 blockade.

Your Answer. d
Correct Answer. d

Copyright 2014 Delhi Academy of Medical Sciences, All Rights Reserved. 30/84
(70). A 69-year-old male with angina develops severe constipation following treatment with

a. Propranolol

b. Captopril

c. Verapamil

d. Dobutamine

Solution. Ans 70: (c) Verapamil.


Reference Read the text below
Sol:
* Constipation,particularly severe with verapamil, may occur with Ca channel blockers.
* In addition, excessive vasodilation may also occur. This can cause dizziness, hypotension, headache, flushing, nausea, and diminished
sensation in fingers and toes.
* Constipation, lethargy, nervousness, and peripheral edema are also seen with the use of Ca channel blockers.

Your Answer. c
Correct Answer. c

(71). Significant relaxation of smooth muscle of both venules and arterioles is produced by which of the following drugs?

a. Hydralazine

b. Minoxidil

c. Diazoxide

d. Sodium nitroprusside

Solution. Ans-71: (d) Sodium nitroprusside


Ref: Read the text below.
Sol:
* Hydralazine, minoxidil,diazoxide, and sodium nitroprusside are all directly acting vasodilators used to treat hypertension. Because
hydralazine, minoxidil, nifedipine, and diazoxide relax arteriolar smooth muscle more than smooth muscle in venules, the effect on
venous capacitance is negligible.
* Sodium nitroprusside,which affects both arterioles and venules, does not increase cardiac output, a feature that enhances the utility of
sodium nitroprusside in the management of hypertensive crisis associated with MI.

Your Answer. a
Correct Answer. d

(72). Physostigmine was therapeutically used for the first time in treatment of glaucoma in 1877 by

a. Laqueur

b. Posselt and Reiman

c. Ahlquist

d. Sir James Black

Solution. Ans-72: (a) Laqueur


Ref.: Read the text below
Sol :
Laqueur
* Posselt and Reiman isolated nicotine from the leaves of Nicotiana tabacum.
* Ahlquist gave the hypothesis that adrenaline acts on different sets of receptors, later named as and .
* Sir James Black synthesized the first -blocker, dichloroisoproterenol, and subsequently, propranolol.

Your Answer. b
Correct Answer. a

Copyright 2014 Delhi Academy of Medical Sciences, All Rights Reserved. 31/84
(73). Which of the following parenteral anticoagulants is a dual coagulation factor inhibitor?

a. Clavatadine

b. Tifacogin

c. Tanogitran

d. Flovagatran

Solution. Ans-73: (c) Tanogitran


Ref.: Read the text below
Sol :

Your Answer. d
Correct Answer. c

(74). According to the recent classification of antitubercular drugs, which of the following drugs is classified under Group II?

a. Ethambutol

b. Streptomycin

c. Levofloxacin

d. Imipenem/Cilastatin

Solution. Ans-74: (b) Streptomycin


Ref.: Read the text below
Sol :

Your Answer. d
Correct Answer. b

Copyright 2014 Delhi Academy of Medical Sciences, All Rights Reserved. 32/84
(75). Glucocorticoids lead to an increase in blood levels of which of the following?

a. Neutrophils

b. Eosinophils

c. Basophils

d. Lymphocytes

Solution. Ans-75: (a) Neutrophils


Ref.: Read the text below
Sol :

Your Answer. a
Correct Answer. a

(76). The treatment of early stage of East African sleeping sickness is

a. Pentamidine

b. Eflornithine

c. Suramin

d. Melarsopol

Solution. Ans-76: (c) Suramin


Ref.: Read the text below
Sol :

Your Answer. b
Correct Answer. c

(77). A 39-year-old pregnant female requires heparin for thromboembolic phenomena. What is the mechanism of action of heparin?

a. Increase in the plasma level of factor IX

b. Inhibition of thrombin and early coagulation steps

c. Inhibition of synthesis of prothrombin and coagulation factors VII, IX, X

d. Inhibition of platelet aggregation in vitro

Solution. Ans-77: (b) Inhibition of thrombin and early coagulation steps


Ref: Read the text below.
Sol:
* Heparin binds to antithrombin III (a plasma protease inhibitor), thereby enhancing its activation.
* The heparinantithrombin III complex interacts with thrombin.
* This inactivates thrombin and other coagulation factors such as VIIa,IXa, Xa, and IIa. Heparin accelerates the rate of thrombin-
antithrombin binding, resulting in the inhibition of thrombin.
* The latter effect is not typically seen with low-molecular-weight heparins that are not of sufficient length to catalyze the inhibition of
thrombin.

Your Answer. c
Correct Answer. b

Copyright 2014 Delhi Academy of Medical Sciences, All Rights Reserved. 33/84
(78). A 42-year-old male with an acute MI is treated with alteplase. What is the mechanism of action of alteplase?

a. Inhibition of platelet thromboxane production

b. Glycoprotein IIb/IIIa antagonist

c. Inhibition of the synthesis of vitamin Kdependent coagulation factors

d. Activation of plasminogen from plasmin

Solution. Ans-78: (d) Activation of plasminogen from plasmin


Ref: Read the text below.
Sol:
* Alteplase is an unmodified tPA. Alteplase activates plasminogen that is bound to fibrin.
* The plasmin that is formed acts directly on fibrin. This results in dissolving the fibrin into fibrin-split products followed by lysis of the
clot.

Your Answer. c
Correct Answer. d

(79). Administration of which of the following antianginal agents results in antianginal effects for only 10 hours, despite detectable therapeutic
plasma levels for 24 hours?

a. Atenolol

b. Transdermal nitroglycerin

c. Amlodipine

d. Amyl nitrite

Solution. Ans-79: (b) Transdermal nitroglycerin


Ref: Read the text below.
Sol:
* Significant tolerance to nitroglycerin develops.
* Transdermal patches can produce therapeutic drug levels for 24 hours, but its effectiveness lasts between 8 and 10 hours.
* A nitrate-free period of at least eight hours is necessary to prevent tolerance.
* Amyl nitrite is inhalable, and its action lasts no longer than five minutes.
* Patients on atenolol and amlodipine do not develop tolerance to these agents.

Your Answer. d
Correct Answer. b

Copyright 2014 Delhi Academy of Medical Sciences, All Rights Reserved. 34/84
(80). Of the following effects, which is not elicited by activation of the parasympathetic nervous system?

a. Decreased heart rate

b. Increased tone of longitudinal smooth muscles of the intestine

c. Contraction of skeletal muscles

d. Contraction of the detrusor of the urinary bladder

Solution. Ans 80 : (c) Contraction of skeletal muscles


Ref Read the text below
Sol:
* Cholinergic impulses arising from the parasympathetic division of the ANS affect many tissues and organs throughout the body.
* Physiologically, this system is concerned primarily with the functions of energy conservation and maintenance of organ function during
periods of reduced activity.
* Slowed heart rate,reduced blood pressure, increased GI motility, emptying of the urinary bladder, and stimulation of secretions from
the pancreas, salivary glands, lacrimal glands, and bronchial and nasopharyngeal glands are all effects observed that are due to
activation of this nervous system.
* However, skeletal muscle contraction is mediated through activation of the somatic nervous system, not the ANS.

Your Answer. c
Correct Answer. c

(81). A 65-year-old diabetic male with erectile dysfunction would be best treated with which of the following?

a. Sildenafil

b. Gossypol

c. Androstenedione

d. Finasteride

Solution. Ans-81: (a) Sildenafil


Reference Read the text below
Sol:
* Sildenafil inhibits the action of phosphodiesterase isoform 5, the major phosphodiesterase in the corpus cavernosum that degrades
cGMP. Nitric oxide is released on sexual stimulation.
* This leads to an increase in cGMP. By inhibiting the enzyme, sildenafil causes the accumulation of cGMP.
* This results in vascular smooth-muscle dilation, allowing increased blood flow into the penis followed by erection.

Your Answer. a
Correct Answer. a

(82). The pharmacokinetic value that most reliably reflects the amount of drug reaching the target tissue after oral administration is the

a. Peak blood concentration

b. Time to peak blood concentration

c. Product of the Vd and the first-order rate constant

d. Area under the blood concentration-time curve (AUC)

Solution. Ans 82: (d) Area under the blood concentration-time curve (AUC)
Ref Read the text below
Sol:
* The fraction of a drug dose absorbed after oral administration is affected by a wide variety of factors that can strongly influence the
peak blood levels and the time to peak blood concentration.
* The Vd and the total body clearance (Vd first-order ke) also are important in determining the amount of drug that reaches the target
tissue.
* Only the area under the blood concentration-time curve, however, reflects absorption, distribution, metabolism, and excretion factors;
it is the most reliable and popular method of evaluating bioavailability.

Your Answer. c
Correct Answer. d

Copyright 2014 Delhi Academy of Medical Sciences, All Rights Reserved. 35/84
(83). What are the most common adverse effects of anticonvulsive drugs?

a. Headache and dizziness

b. Gastrointestinal symptoms

c. Alternation of cognition and mentation

d. All of the above

Solution. Ans-83: (d) All of the above


Reference Read the text below
Sol:
* Alternation in cognition and mentation, gastrointestinal symptoms, appetite and body weight, and headache or dizziness are all
common adverse effects of anticonvulsive drugs.

Your Answer. a
Correct Answer. d

(84). For a drug that obeys first order(linear) kinetics and fits a one compartment model of elimination:

a. Its rate of elimination is proportional to its plasma concentration

b. Following cessation of an intravenous infusion the plasma concentration increases exponentially

c. The half-life is proportional to the dose

d. the half-life is unaffected by renal function

Solution. Ans 84: (a) Its rate of elimination is proportional to its plasma concentration
Ref Read the text below
Sol:
* Although the one compartment model is an oversimplification.
* Once absorption and distribution are complete many drugs do obey first order elimination kinetics

Your Answer. d
Correct Answer. a

Copyright 2014 Delhi Academy of Medical Sciences, All Rights Reserved. 36/84
(85). Medical management for detrusor instability ?

a. Duloxetine

b. Flavoxate

c. Toletradine

d. Solarifinacin

Solution. Ans 85: (c) Toletradine


Ref Campbell urology ,GG1 195,196, KDT 95,110
Sol:

* Detrusor overactivity is a generic term for involuntary detrusor contractions. The ICS defined overactive detrusor function as
involuntary detrusor contractions during the filling phase, which may be spontaneous or provoked and which the patient cannot
completely suppress .
* The term detrusor overactivity should be used when the etiology of the involuntary detrusor contractions is unclear. Involuntary
detrusor contractions that are due to neurologic disorders are defined as detrusor hyperreflexia, whereas the term detrusor instability
should be use to denote involuntary detrusor contractions that are not due to neurologic disorders.
* Anticholinergics are competitive inhibitors of acetylcholine that block the muscarinic effects. If fully effective, these drugs would result
in abolition of the involuntary detrusor contraction.Propantheline bromide is the prototype anticholinergic, but in current practice
tolterodine and oxybutynin chloride are the two most widely used anticholinergics.
* Overactive urinary bladder disease can be successfully treated with muscarinic antagonists, primarily Tolterodine(goodman gilman ch
7)
* Stress urinary incontinence has been treated with some success with duloxetine (YENTREVE), which acts centrally to influence 5-HT
and NE levels.

Your Answer. b
Correct Answer. c

Copyright 2014 Delhi Academy of Medical Sciences, All Rights Reserved. 37/84
(86). Of the following, which is unlikely to be associated with drug distribution into and out of the central nervous system (CNS)?

a. Most drugs enter the CNS by simple diffusion at rates that are proportional to the lipid solubility of the nonionized form of the drug

b. Receptor-mediated transport allows certain peptides to gain access to the brain

c. Strongly ionized drugs freely enter the CNS through carrier-mediated transport systems

d. Some drugs leave the CNS by passing from the cerebrospinal fluid into the dural blood sinuses through the arachnoid villi

Solution. Ans 86: (c) Strongly ionized drugs freely enter the CNS through carrier-mediated transport systems
Ref Read the text below
Sol:
* Drugs can enter the brain from the circulation by passing through the blood-brain barrier.
* This boundary consists of several membranes, including those of the capillary wall, the glial cells closely surrounding the capillary, and
the neuron.
* In most cases, lipid-soluble drugs diffuse through these membranes at rates that are related to their lipid-to-water partition coefficients.
Therefore, the greater the lipid solubility of the nonionized fraction of a weak acid or base, the more freely permeable the drug is to the
brain. Some drugs enter the CNS through specific carrier-mediated or receptor-mediated transport processes.
Carrier-mediated systems appear to be involved predominantly in the transport of a variety of nutrients through the blood-brain barrier;
however, the thyroid hormone 3,5,3'-triiodothyronine and drugs such as levodopa and methyldopa, which are structural derivatives of
phenylalanine, cross the blood-brain barrier via carrier-mediated transport. Receptor-mediated transport functions to permit a peptide
(e.g., insulin) to enter the CNS; therefore, some peptide-like drugs are believed to gain access to the brain by this mechanism.
Regardless of the process by which drugs can enter the CNS, strongly ionized drugs (e.g., quaternary amines) are unable to enter the
CNS from the blood. The exit of drugs from the CNS can involve (1) diffusion across the blood-brain barrier in the reverse direction at
rates determined by the lipid solubility and degree of ionization of the drug, (2) drainage from the cerebrospinal fluid (CSF) into the
dural blood sinuses by flowing through the wide channels of the arachnoid villi, and (3) active transport of certain organic anions and
cations from the CSF to blood across the choroid plexuses.

Your Answer. c
Correct Answer. c

(87). All of the following are examples of prodrugs except :

a. Levodopa

b. Azathioprine

c. Benorylate

d. Frusemide

Solution. Ans 87: (d) Frusemide


Ref Read the text below
Sol:
* Levodopa dopamine
* Azathioprine 6-mercaptopurine
* Benorylate paracetamol + aspirin

Your Answer. d
Correct Answer. d

Copyright 2014 Delhi Academy of Medical Sciences, All Rights Reserved. 38/84
(88). A 46-year-old man is admitted to the emergency department. He has taken more than ninety digoxin tablets (0.25 mg each), ingesting
them about three hours after admission. His pulse is fifty to sixty beats per minute, and the electrocardiogram shows third-degree heart
block. Which one of the following is the most important therapy to initiate in this patient?

a. Digoxin immune fab

b. Potassium salts

c. Lidocaine

d. Phenytoin

Solution. Ans-88: (a) Digoxin immune fab


Reference Read the text below
Sol:
* In the severely poisoned patient, reduction of digoxin plasma concentrations in paramount, and can be accomplished with
administration of antidigoxin antibodies.
* Potassium concentrations, if low, can be increased, but to not much greater than 4 mM.
Antiarrhythmics are useful if there is need, but not in this case. DC cardioversion is only used if ventricular fibrillation occurs.

Your Answer. c
Correct Answer. a

(89). Which of the following agents should be considered for additional antimicrobial therapy with ciprofloxacin in the setting of bioterrorist-
related inhalational anthrax?

a. Ceftazidime

b. Aztreonam

c. Vancomycin

d. Trimethoprim-sulfamethoxazole

Solution. Ans 89: (c) Vancomycin


Ref Read the text below
Sol:
* B. anthracis has natural resistance to sulfa drugs, trimethoprim, aztreonam, and third-generation cephalosporins (such as ceftriaxone
and ceftazidime).
* Antimicrobial agents that should be considered in addition to ciprofloxacin or doxycycline in the treatment of inhalational anthrax
include penicillin or ampicillin,vancomycin, rifampin, chloramphenicol, clindamycin, imipenem,and clarithromycin.

Your Answer. d
Correct Answer. c

(90). A medication that inhibits the hepatic metabolism of warfarin

a. Cimetidine

b. Aspirin

c. Indometacin

d. Amoxicillin

Solution. Ans 90: (a) Cimetidine


Ref Read the text below
Sol:
* Interactions of cimetidine with warfarin are clinically significant as this compound possesses a narrow therapeutic index.
* The combined use of warfarin and cimetidine results in prolongation of prothrombin time and increased bleeding risk. Studies have
documented that cimetidine inhibits the hepatic metabolism of warfarin.
* The cimetidine-warfarin interaction appears to be dose-related and is associated with considerable interpatient variability.
* Even though clinically significant drug interactions have been observed between warfarin and cimetidine, concurrent use is not an
absolute contraindication.

Your Answer. a
Correct Answer. a

Copyright 2014 Delhi Academy of Medical Sciences, All Rights Reserved. 39/84
(91). According to the CDC guidelines for the appropriate use of vancomycin, which of the following cases would be considered appropriate
use?

a. Primary treatment of antibiotic-associated colitis

b. Continued use in a patient with methicillin-sensitive S. epidermidis

c. Once a week treatment in a hemodialysis patient with a streptococcal infection

d. Methicillin-sensitive S. aureus endocarditis in a patient with a history of anaphylaxis to penicillin

Solution. Ans 91: (d) Methicillin-sensitive S. aureus endocarditis in a patient with a history of anaphylaxis to penicillin
Ref Read the text below
Sol:
The following are CDC guidelines for the prevention of VRE: Situations in which the use of vancomycin is appropriate:
* Treatment of serious infections due to -Iactam-resistant gram-positive organisms
* Treatment of serious infections due to gram-positive organisms in patients with serious -Iactam allergies
* Treatment of antibiotic-associated colitis (AAC) when treatment with metronidazole has failed or if the AAC is potentially life
threatening
* Prophylaxis for endocarditis for certain procedures based on American Heart Association recommendations
* Prophylaxis for certain surgical procedures involving implantation of prosthetic materials in hospitals with a high rate of MRSA or
MRSE.

Your Answer. a
Correct Answer. d

(92). Antimicrobial prophylaxis following a significant exposure to anthrax spores should be continued for how long?

a. 2 days

b. 12 days

c. 45 days

d. 60 days

Solution. Ans 92: (d) 60 days


Ref Read the text below
Sol:
* Given that the incubation period for inhalational anthrax is between 2 and 43 days, antimicrobial prophylaxis should be continued for
60 days to prevent germination of the spores and subsequent disease.

Your Answer. b
Correct Answer. d

(93). Which antibiotic is least likely to cause C. difficile colitis?

a. Clindamycin

b. Ceftriaxone

c. Metronidazole

d. Cefoxitin

Solution. Ans 93: (c) Metronidazole


Ref Read the text below
Sol:
* Antibiotics that kill off normal flora but allow C. difficile to survive create ecological conditions for C. difficile overgrowth.
* Because metronidazole kills C. difficile, it is less likely to give rise to the infection than other antibiotics.
* Clindamycin and kill a broad spectrum of bacteria but do not harm C. difficile; these antibiotics are associated with some of the highest
rates of pseudomembranous colitis

Your Answer. a
Correct Answer. c

Copyright 2014 Delhi Academy of Medical Sciences, All Rights Reserved. 40/84
(94). Which antibiotic is a reasonable choice for a healthy young woman diagnosed with an uncomplicated lower urinary tract infection?

a. Imipenem

b. Azithromycin

c. Nitrofurantoin

d. Trovafloxacin

Solution. Ans 94: (c) Nitrofurantoin


Ref - Read the text below
Sol:
* Nitrofurantoin is a useful antibiotic because it works selectively in the bladder, after being concentrated in the urine.Because of this
selective action, it is not appropriate for patients with pyelonephritis, as it will not kill bacteria in the bloodstream or kidneys.
* The drug will also not work in patients with renal failure, whose kidneys do not concentrate the drug. Whereas quinolones such as
ciprofloxacin or levofloxacin are often used for UTls, trovafloxacin would not be used because it can cause serious liver toxicity.
Imipenem is a parenteral, extremely broad spectrum antibiotic that would never be used in the outpatient setting.
* Azithromycin is rarely used for UTls because its spectrum and achievable drug levels do not match well with urinary tract pathogens. It
is more often used for respiratory and ENT infections.
* Metronidazole is used to treat vaginal infections (bacterial vaginosis or trichomonas), but UTls are usually due to aerobic bacteria not
killed by this drug.

Your Answer. c
Correct Answer. c

(95). If a drug is repeatedly administered at dosing intervals that are equal to its elimination half-life, the number of doses required for the
plasma concentration of the drug to reach the steady state is

a. 2 to 3

b. 4 to 5

c. 6 to 7

d. 8 to 9

Solution. Ans 95: (b) 4 to 5.


Reference Read the text below
Sol:
* When a drug is administered in multiple doses and each dose is given prior to the complete elimination of the previous dose, the mean
plasma concentration (C) of the drug during each dose interval rises.
* The plasma concentration will continue to rise until it reaches a plateau, or steady state. At this time, the plasma concentration will
fluctuate between a maximum (Cmax) and a minimum (Cmin) level, but, more important, the amount of drug eliminated per dose interval
will equal the amount of drug absorbed per dose.
* When a drug is given at a dosing interval that is equal to its elimination half-life, it will reach 50% of its steady-state plasma
concentration after one half-life, 75% after two half-lives, 87.5% after three, 93.75% after four, and 96.87% after five.
* Thus, from a practical viewpoint, regardless of the magnitude of the dose or the half-life, the steady state will be achieved in four to five
half-lives.

Your Answer. b
Correct Answer. b

Copyright 2014 Delhi Academy of Medical Sciences, All Rights Reserved. 41/84
(96). Which of the following is classified as belonging to the tyrosine kinase family of receptors?

a. GABA A receptor

b. -adrenergic receptor

c. Insulin receptor

d. Nicotinic II receptor

Solution. Ans 96: (c) Insulin receptor. Reference Read the text below Sol: There are four major classes of receptors:
* Ion channel receptors,
* Receptors coupled to G proteins,
* Receptors with tyrosine-specific kinase activity, and
* Nuclear receptors.
* In most cases, drugs that act via receptors do so by binding to extracellular receptors that transduce the information intracellularly by
a variety of mechanisms.
* Activated ion channel receptors enhance the influx of extracellular ions into the cell; for example, the nicotinic-II cholinergic receptor
selectively opens a channel for sodium ions and the GABAA receptor functions as an ionophore for chloride ions.
* Receptors coupled to guanine nucleotide-binding proteins (G proteins) act either by opening an ion channel or by stimulating or
inhibiting specific enzymes (e.g., -adrenergic receptor stimulation leads to an increase in cellular adenylate cyclase activity).
* When stimulated, receptors with tyrosine-specific protein kinase activity activate this enzyme to enhance the transport of ions and
nutrients across the cell membrane; for example, insulin receptors function in this manner and increase glucose transport into insulin-
dependent tissues.

Your Answer. b
Correct Answer. c

(97). Of the following characteristics, which is unlikely to be associated with the process of facilitated diffusion of drugs?

a. The transport mechanism becomes saturated at high drug concentrations

b. The process is selective for certain ionic or structural configurations of the drug

c. If two compounds are transported by the same mechanism, one will competitively inhibit the transport of the other

d. The drug crosses the membrane against a concentration gradient and the process requires cellular energy

Solution. Ans 97: (d) The drug crosses the membrane against a concentration gradient and the process requires cellular energy.
Reference Read the text below
Sol:
* Drugs can be transferred across biologic membranes by passive processes (i.e., filtration and simple diffusion) and by specialized
processes (i.e., active transport, facilitated diffusion,and pinocytosis). Active transport is a carrier-mediated process that shows all of the
characteristics listed in the question.
* Facilitated diffusion is similar to active transport except that the drug is not transported against a concentration gradient and no
energy is required for this carrier-mediated system to function.
* Pinocytosis usually involves transport of proteins and macromolecules by a complex process in which a cell engulfs the compound
within a membrane-bound vesicle.

Your Answer. b
Correct Answer. d

Copyright 2014 Delhi Academy of Medical Sciences, All Rights Reserved. 42/84
(98). Of the following, which is a phase II biotransformation reaction?

a. Sulfoxide formation

b. Nitro reduction

c. Ester hydrolysis

d. Sulfate conjugation

Solution. Ans 98: (d) Sulfate conjugation.


Reference Read the text below
Sol:
* Biotransformation reactions involving the oxidation, reduction, or hydrolysis of a drug are classified as phase I (or nonsynthetic)
reactions; these chemical reactions may result in either the activation or inactivation of a pharmacologic agent.
* There are many types of these reactions; oxidations are the most numerous. Phase II (or synthetic) reactions, which almost always
result in the formation of an inactive product, involve conjugation of the drug (or its derivative) with an amino acid, carbohydrate,
acetate, or sulfate.
* The conjugated form(s) of the drug or its derivatives may be more easily excreted than the parent compound.

Your Answer. d
Correct Answer. d

(99). In the treatment of bacterial meningitis in children, the drug of choice is

a. Penicillin G

b. Penicillin V

c. Erythromycin

d. Ceftriaxone

Solution. Ans 99 : (d) Ceftriaxone


Ref Read the text below
Sol:
* Penicillins were used in the treatment of meningitis because of their ability to pass across an inflamed blood-brain barrier.
* The third-generation cephalosporin, ceftriaxone,is preferred because it is effective against -lactamase producing strains of H.
influenzae that may cause meningitis in children.

Your Answer. d
Correct Answer. d

(100). In patients with hepatic coma, decreases in the production and absorption of ammonia from the gastrointestinal (GI) tract will be
beneficial. The antibiotic of choice in this situation would be

a. Neomycin

b. Tetracycline

c. Penicillin G

d. Chloramphenicol

Solution. Ans 100 : (a) Neomycin


Ref Read the text below
Sol:
* Neomycin, an aminoglycoside,is not significantly absorbed from the GI tract.
* After oral administration, the intestinal flora is suppressed or modified and the drug is excreted in the feces.
* This effect of neomycin is used in hepatic coma to decrease the coliform flora, thus decreasing the production of ammonia that causes
the levels of free nitrogen to decrease in the bloodstream.
* Other antimicrobial agents (e.g., tetracycline, penicillin G, chloramphenicol, and cephalothin) do not have the potency of neomycin in
causing this effect

Your Answer. a
Correct Answer. a

Copyright 2014 Delhi Academy of Medical Sciences, All Rights Reserved. 43/84
(101). Of the following, which is unlikely to be associated with the binding of drugs to plasma proteins?

a. Acidic drugs generally bind to plasma albumin; basic drugs preferentially bind to 1-acidic glycoprotein

b. Plasma protein binding is a reversible process

c. The fraction of the drug in the plasma that is bound is inactive and generally unavailable for systemic distribution

d. Plasma protein binding generally limits renal tubular secretion and biotransformation

Solution. Ans 101: (d) Plasma protein binding generally limits renal tubular secretion and biotransformation.
Reference Read the text below
Sol:
* Because only the free (unbound) fraction of a drug can cross biologic membranes, binding to plasma proteins limits a drugs
concentration in tissues and, therefore, decreases the apparent Vd of the drug.
* Plasma protein binding will also reduce glomerular filtration of the drug because this process is highly dependent on the free drug
fraction.
* Renal tubular secretion and biotransformation of drugs are generally not limited by plasma protein binding because these processes
reduce the free drug concentration in the plasma.
* If a drug is avidly transported through the tubule by the secretion process or is rapidly biotransformed, the rates of these processes
may exceed the rate of dissociation of the drug-protein complex (in order to restore the free:bound drug ratio in plasma) and, thus,
become the rate-limiting factor for drug elimination.
* This assumes that equilibrium conditions exist and that other influences (e.g., changes in pH or the presence of other drugs) do not
occur.

Your Answer. a
Correct Answer. d

(102). Of the following, which is unlikely to be associated with drug distribution into and out of the central nervous system (CNS)?

a. The blood-brain barrier, which involves drug movement through glial cell membranes as well as capillary membranes, is the main
hindrance to drug distribution to the CNS

b. Most drugs enter the CNS by simple diffusion at rates that are proportional to the lipid solubility of the nonionized form of the drug

c. Receptor-mediated transport allows certain peptides to gain access to the brain

d. Strongly ionized drugs freely enter the CNS through carrier-mediated transport systems

Solution. Ans 102: (d) Strongly ionized drugs freely enter the CNS through carrier-mediated transport systems.
Reference Read the text below
Sol:
* Drugs can enter the brain from the circulation by passing through the blood-brain barrier. This boundary consists of several
membranes, including those of the capillary wall, the glial cells closely surrounding the capillary, and the neuron.
* In most cases, lipidsoluble drugs diffuse through these membranes at rates that are related to their lipid-to-water partition coefficients.
Therefore, the greater the lipid solubility of the nonionized fraction of a weak acid or base, the more freely permeable the drug is to the
brain. Some drugs enter the CNS through specific carrier-mediated or receptor-mediated transport processes.
* Carrier mediated systems appear to be involved predominantly in the transport of a variety of nutrients through the blood-brain
barrier; however, the thyroid hormone 3,5,3-triiodothyronine and drugs such as levodopa and methyldopa,which are structural
derivatives of phenylalanine, cross the bloodbrain barrier via carrier-mediated transport.
* Receptor-mediated transport functions to permit a peptide (e.g., insulin) to enter the CNS; therefore,some peptide-like drugs are
believed to gain access to the brain by this mechanism.
* Regardless of the process by which drugs can enter the CNS,strongly ionized drugs (e.g., quaternary amines) are unable to enter the
CNS from the blood.
* The exit of drugs from the CNS can involve (1) diffusion across the blood-brain barrier in the reverse direction at rates determined by
the lipid solubility and degree of ionization of the drug, (2) drainage from the cerebrospinal fluid (CSF) into the dural blood sinuses by
flowing through the wide channels of the arachnoid villi, and (3) active transport of certain organic anions and cations from the CSF to
blood across the choroids plexuses.

Your Answer. a
Correct Answer. d

Copyright 2014 Delhi Academy of Medical Sciences, All Rights Reserved. 44/84
(103). The mechanism of action by which pyrantel pamoate is effective for the treatment of Necator americanus (hookworm) disease is

a. Interference with cell-wall synthesis

b. Interference with cell division

c. Inhibition of neuromuscular transmission

d. Interference with protein synthesis

Solution. Ans 103 : (c) Inhibition of neuromuscular transmission


Ref Read the text below
Sol:
* Pyrantel pamoate is an antihelminthic that acts primarily as a depolarizing neuromuscular blocker.
* In certain worms, a spastic neuromuscular paralysis occurs, resulting in the expulsion of the worms from the intestinal tract of the host.
* Pyrantel also exerts its effect against parasites via release of acetylcholine and inhibition of cholinesterase.

Your Answer. a
Correct Answer. c

(104). Vertigo, inability to perceive termination of movement, and difficulty in sitting or standing without visual clues are some of the toxic
reactions that are likely to occur in about 75% of patients treated with

a. Penicillin G

b. Doxycycline

c. Amphotericin B

d. Streptomycin

Solution. Ans 104 : (d) Streptomycin


Ref Read the text below
Sol:
* Streptomycin and other aminoglycosides can elicit toxic reactions involving both the vestibular and auditory branches of the eighth
cranial nerve.
* Patients receiving an aminoglycoside should be monitored frequently for any hearing impairment owing to the irreversible deafness
that may result from its prolonged use.
* None of the other agents listed in the question adversely affect the function of the eighth cranial nerve.

Your Answer. d
Correct Answer. d

(105). Which of the following is unlikely to be associated with oral drug administration of an enteric-coated dosage form ?

a. Irritation to the gastric mucosa with nausea and vomiting

b. Destruction of the drug by gastric acid or digestive enzymes

c. Unpleasant taste of the drug

d. Variability in absorption caused by fluctuations in gastric emptying time

Solution. Ans 105: (d) Variability in absorption caused by fluctuations in gastric emptying time. Reference Read the text below
Sol:
* Tasteless enteric-coated tablets and capsules are formulated to resist the acidic pH found in the stomach.
* Once the preparation has passed into the intestine, the coating dissolves in the alkaline milieu and releases the drug.
* Therefore, gastric irritation, drug destruction by gastric acid, and the forming of complexes of the drug with food constituents will be
avoided.

Your Answer. c
Correct Answer. d

Copyright 2014 Delhi Academy of Medical Sciences, All Rights Reserved. 45/84
(106). Which toxicity do all of the first-line TB drugs have in common?

a. Neuropathy

b. Optic neuritis

c. Hepatotoxicity

d. Serious drug-drug interactions

Solution. Ans 106: (c) Hepatotoxicity


Ref Read the text below
Sol:
* All the first-line drugs can cause hepatotoxicity, the major toxicity of TB therapy. INH can cause neuropathy (which can be prevented
by co-administration of vitamin B 6 ), ethambutol can cause optic neuritis, rifampin can cause serious drug-drug interactions (particularly
with HIV protease inhibitors), and pyrazinimide causes hyperuricemia (although this is usually asymptomatic).

Your Answer. c
Correct Answer. c

(107). Which drug is currently approved for administration once a week as prophylaxis for MAC in HIV infection?

a. Rifabutin

b. Azithromycin

c. Clarithromycin

d. Ethambutol

Solution. Ans 107: (b) Azithromycin


Ref Read the text below
Sol:
The various prophylactic regimens for MAC in HIV infection (usually initiated at CD4 counts :5 50 cells/fLL) include azithromycin 1200
mg orally every week, clarithromycin 500 mg orally twice a day, and rifabutin 300 mg orally every day.

Your Answer. b
Correct Answer. b

(108). Which of the following cephalosporins would have increased activity against anaerobic bacteria such as Bacteroides fragilis?

a. Cefaclor

b. Cephalothin

c. Cephalexin

d. Cefoxitin

Solution. Ans 108 : (d) Cefoxitin


Ref Read the text below
Sol:
* Cefoxitin and cefmetazole are suitable for treating intraabdominal infections.
* Such infections are caused by mixtures of aerobic and anaerobic Gram-negative bacteria like B. fragilis. Cefoxitin alone has been shown
to be as effective as the traditional therapy of clindamycin plus gentamicin.

Your Answer. d
Correct Answer. d

Copyright 2014 Delhi Academy of Medical Sciences, All Rights Reserved. 46/84
(109). An epileptic on hypertensive treatment develops acute hemorrhagic pancreatitis. Which medication is the most likely cause ?

a. Propranolol

b. Nifedipine

c. Verapamil

d. Sodium Valproate

Solution. Ans 109: (d) Sodium Valproate


Ref Read the text below
Sol:
* Valproate (VPA)-associated pancreatitis appears to be more frequent in young persons (mean age 16.4 years) but may occur at any age.
* The highest risk appears to exist during the first months of treatment: 43.8% of the cases developed during the first 3 months, and
68.8% developed during the first year. In most patients, the reaction was rapidly reversible when valproate was discontinued. It could be
severe with deaths reported.
* Asymptomatic elevation of serum amylaseis seen in 10%. Awareness of the problem and early discontinuation of VPA may be effective
in preventing serious reactions.

Your Answer. d
Correct Answer. d

(110). A 24 year old following her PhD in chemistry is working in a laboratory when a mercury-containing solvent was accidentally spilled over
her desk top. She inhaled the vapour. She started coughing and soon after was severely nauseated .She was urgently admitted to the
emergency department complaining of a tight feeling in her chest, and having difficulty with breathing .Which is the best drug treatment
for mercury poisoning?

a. Penicillamine

b. Hydrocortisone

c. Subcutaneous Adrenalin

d. Glyceryltrinitrate

Solution. Ans 110: (a) Penicillamine


Ref Read the text below
Sol:
a. Penicillamine is also indicated for the treatment of copper, arsenic, lead and zinc poisoning, Wilson's disease and cystinuria Mercury
may still be an occupational hazard for people working in medical care facilities.

Your Answer. c
Correct Answer. a

Copyright 2014 Delhi Academy of Medical Sciences, All Rights Reserved. 47/84
(111). A 38 year old woman being treated for hypertension and diabetes has the sudden onset of swelling and tenderness of the wrists and
knees. On examination she is febrile and flushed. A friction rub can be heard at the left lower sternal border.
Which of the following drugs is most likely the cause of these findings?

a. Metformin

b. Hydralazine

c. Minoxidil

d. Nitroprusside

Solution. Ans 111: (b) Hydralazine


Ref - Read the text below
Sol:
* Drug-induced lupus refers to a form of lupus caused by medication. It causes some symptoms similar to those of SLE (arthritis, rash,
fever, and chest pain, but not kidney disease) that go away when the drug is stopped.
* Common medications that may cause drug-induced lupus include hydralazine (Apresoline), procainamide (Procan, Pronestyl),
methyldopa (Aldomet), quinidine (Quinaglute), isoniazid (INH), and some anti-seizure medications such as phenytoin (Dilantin) or
carbamazepine (Tegretol).
* This complication is more common in femlaes. It is also more common in 'slow acetylators' since the metabolism of hydralazine is
reduced in these patients.

Your Answer. b
Correct Answer. b

(112). A patient has muscular weakness secondary to autoantibodies against acetylcholine (ACh) receptor proteins on the cell surfaces of motor
end plates. This condition may be completely relieved for a period of two hours by adminstration of :

a. Intravenous cyclosporine

b. Oral antihistamine

c. Intravenous anticholinesterase

d. Oral pyridostigmine

Solution. Ans 112: (d) Oral pyridostigmine


Ref Read the text below
Sol:
* The blood of most patients with myasthenia gravis contains autoantibodies against acetylcholine (ACh) receptor proteins on the cell
surfaces of the motor end plates etc.
* The autoantibody competes for the ACh receptor and inhibits synaptic transmission, so muscular contraction is greatly inhibited.
Deposition of immune complexes eventually destroys the ACh-receptor protein. Intravenous injection of an anticholinesterase improves
the muscle strength immediately, but the beneficial effect is gone within 3 min.
* Thymectomy improves the condition and the prognosis also in the group of patients without thymoma.
* Oral anticholinesterase (such as pyridostigmine) has beneficial effect over 2-4 hours. They inhibit the enzyme acetylcholine-esterase,
and thereby prolong the effect of naturally occurring acetylcholine on the receptors. In severe cases this treatment is inefficient, and
immune-suppressants such as corticosteroids are sometimes favourable.

Your Answer. d
Correct Answer. d

Copyright 2014 Delhi Academy of Medical Sciences, All Rights Reserved. 48/84
(113). A 34-year-old male with Hodgkins disease is treated with the adriamycin,bleomycin, vinblastine, and decarbazine (ABVD) regimen. What
is the mechanism of action of vinblastine?

a. Scission of DNA strands

b. Inhibition of dihydrofolate reductase

c. Inhibition of enzymes involved in purine metabolism

d. Prevention of assembly of tubulin dimers into microtubules

Solution. Ans 113: (d) Prevention of assembly of tubulin dimers into microtubules
Reference Read the text below
Sol:
* Vinblastine binds to tubulin and blocks the protein from polymerizing to microtubules.
* The drug-tubulin complex binds to the developing microtubule,resulting in inhibition of microtubule assembly and subsequent
depolymerization.

Your Answer. d
Correct Answer. d

(114). A 60-year-old male with hematuria is found to have a small localized tumor of the bladder that is diagnosed as a carcinoma. Which of the
following agents should be given intravesicularly?

a. Allopurinol

b. Asparaginase

c. Methotrexate

d. BCG vaccine

Solution. Ans 114: (d) BCG vaccine


Reference Read the text below
Sol:
* Bacille Calmette-Gurin vaccine is a nonspecific stimulant of the reticuloendothelial system.
* It is an attenuated strain of Mycobacterium bovis that appears most effective in small,localized bladder tumors.
* This agent is approved for intravesicular use in bladder cancer. Adverse reactions are associated with the renal system, such as
problems with urination, infection, and cystitis.

Your Answer. d
Correct Answer. d

(115). A 45-year-old male has an insulinoma. Which of the following agents is the treatment of choice?

a. Cyclophosphamide

b. Carboplatin

c. Vincristine

d. Streptozocin

Solution. Ans 115: (d) Streptozocin


Reference - Read the text below
Sol:
* Streptozocin is an alkylating agent with the capacity to cross-link DNA, thereby inhibiting its synthesis.
* It is a nitrosourea-like antibiotic that contains a glucosamine moiety that allows it to be selectively taken up by the ] cells of the islets
of Langerhans. Consequently, it can be useful in treating metastatic islet cell carcinoma.

Your Answer. d
Correct Answer. d

Copyright 2014 Delhi Academy of Medical Sciences, All Rights Reserved. 49/84
(116). A medication that is associated with gingivitis

a. Phenytoin

b. Carbamazepine

c. Propranolol

d. Diazepam

Solution. Ans 116: (a) Phenytoin


Ref Read the text below
Sol:
* Gingivitis is a form of periodontal disease.
* Periodontal disease is when inflammation and infection destroy the tissues that support the teeth, including the gingiva (gums), the
periodontal ligaments, and the tooth sockets (alveolar bone )

Your Answer. a
Correct Answer. a

(117). Vascular abnormalities are a known side effect of several cytotoxic drugs. Which of the following cytotoxic drugs is most likely to lead to
thrombotic micro-angiopathy ?

a. Mitomycin

b. 5-fluorouracil

c. Taxoids

d. Cyclopentenyl cytosine

Solution. Ans 117: (a) Mitomycin


Ref Read the text below
Sol:
* The term 'thrombotic microangiopathy' (TMA) describes syndromes of microangiopathic hemolytic anemia, thrombocytopenia, and
variable signs of organ impairment, due to platelet aggregation in the microcirculation.
* Mitomycin-C is, etiologically, the most common causative agent inducing 'thrombotic microangiopathy' in a dose dependent manner.

Your Answer. a
Correct Answer. a

(118). A fifty-five year old with chronic cardiac failure is on treatment with digoxin and a loop diuretic.This combination is likely to cause
digoxin toxicity by the following mechanism:

a. This combination increases the half-life of digoxin

b. Diuretics decrease potassium levels

c. Frusemide and digoxin interact to form a poisonous compound

d. Digoxin is a competitive inhibitor of frusemide

Solution. Ans 118: (b) Diuretics decrease potassium levels


Ref Read the text below
Sol:
* Loop and thiazide diuretics decrease potassium and magnesium levels, predisposing patients taking both a diuretic and digoxin to an
increased risk of digoxin toxicity.
* Also, amphotericin B (Fungizone), an antifungal, has an additive potassium-lowering effect when given with a thiazide or loop diuretic.
* Thiazides may increase the blood levels of lithium. Bile acid sequestrants cholestyramine (Questran) and colestipol (Colestid) decrease
the absorption of thiazide diuretics when given concomitantly, while nonsteroidal anti-inflammatory drugs such as indomethacin
(Indocin) may decrease the therapeutic effects of both the loop and thiazide diuretics.

Your Answer. b
Correct Answer. b

Copyright 2014 Delhi Academy of Medical Sciences, All Rights Reserved. 50/84
(119). Diplopia and ataxia are the most common dose-related adverse effects of :

a. Vigabatrin

b. Carbamazepine

c. Gabapentin

d. Topiramate

Solution. Ans 119: (b) Carbamazepine


Ref Read the text below
Sol: Carbamazepine
* The most common side effects are dizziness, drowsiness, ataxia, diplopia, nystagmus (around 50%), heachache, tremor (often responds
to propranolol), cognitive blunting, decreased libido in men (through lowering of free testosterone levels secondary to enzyme induction),
hyponatremia & water intoxication, sinus node dysfunction in combination with lithium, lowered response of TSH to TRH.

Your Answer. c
Correct Answer. b

(120). A 40 year old has repeated episodes of depressed mood in response to feeling rejected, and a craving for sweets and chocolate. These
reactive mood changes are accompanied by hypersomnia, lethargy and increased appetite, particularly with a preference for
carbohydrates. The most appropriate treatment would be

a. Selective Serotonin Re-uptake Inhibitors

b. Tricyclic antidepressants

c. Lithium

d. Monoamine Oxidase Inhibitors

Solution. Ans 120: (d) Monoamine Oxidase Inhibitors


Ref Read the text below
Sol:
* Monoamine Oxidase Inhibitors are effective treatments in patients with major depressive disordee with atypical features such as inthis
case.
* At least three categories of atypical depression have been described. The hysteroid dysphoria is characterized by repeated episodes of
depressed mood in response to feeling rejected, and a craving for sweets and chocolate.
* Two other issues are characterized by a cyclical occurrence of changes of mood and appetite, i.e., the late luteal phase dysphoric
disorder (DSM-III-R, appendix), or 'the premenstrual syndrome' (PMS), and the major depression with seasonal pattern (DSM-III-R), or
seasonal affective disorder (SAD).
* The reactive mood changes are frequently accompanied by features as hypersomnia, lethargy and increased appetite, particularly with
a preference for carbohydrates Major Depressive Disorder with atypical features should be treated with monoamine Oxidase Inhibitors.

Your Answer. a
Correct Answer. d

(121). All of the following fluoroquinolones have been withdrawn except

a. Trovafloxacin

b. Gatifloxacin

c. Gemifloxacin

d. Grepafloxacin

Solution. Ans-121: (c) Gemifloxacin


Ref.: Read the text below
Sol :
Gemifloxacin. All others have been withdrawn.
* Trovafloxacin due to hepatotoxicity
* Gatifloxacin due to hypo/hyper glycemia
* Grepafloxacin due to adverse cardiac events

Your Answer. b
Correct Answer. c

Copyright 2014 Delhi Academy of Medical Sciences, All Rights Reserved. 51/84
(122). Which of the following drugs is effective against HIV with Y181C mutation?

a. Festinavir

b. Lersivirine

c. Elvitegravir

d. Dolutegravir

Solution. Ans-122: (b) Lersivirine


Ref.: Read the text below
Sol :
Lersivirine
* Festinavir is a NRTI shown to be effective in adefovir and tenofovir resistance
* Lersivirine is a NNRTI effective against HIV with Y181C mutation
* Elvitegravir is a integrase inhibitor used for initial / nave treatment
* Dolutegravir is a integrase inhibitor approved for raltegravir-resistant infections

Your Answer. Not Attempted


Correct Answer. b

(123). Which of the following is granted orphan drug status for the treatment of Dravets syndrome?

a. Stiripentol

b. Icatibant

c. Pitolisant

d. Tafamidis

Solution. Ans-123: (a) Stiripentol


Ref.: Read the text below
Sol :
Stiripentol
* Stiripentol increases GABA transmission. It is granted orphan drug status for the treatment of Dravets syndrome (Severe myoclonic
epilepsy in infants).
* Icatibant is a bradykinin B2 receptor antagonist. It is used for acute attacks of hereditary angioedema in adults with C1-esterase
inhibitor deficiency.
* Pitolisant (a/k/a Tiprolisant) is a inverse agonist of H3 receptor. It is under trial for schizophrenia and Parkinsons disease.
* Tafamidis stabilizes the correctly folded tetrameric form of TTR. It is used in familial amyloid polyneuropathy.

Your Answer. a
Correct Answer. a

(124). Which of the following statins can be administered at any time during the day?

a. Lovastatin

b. Simvastatin

c. Pravastatin

d. Atorvastatin

Solution. Ans-124: (d) Atorvastatin


Ref.: Read the text below
Sol :
Atorvastatin. Atorvastatin and rosuvastatin can be administered at any time of the day due to long half-lives (>12 hours).
All other statins have a short half-life (<4 hours), hence given just before bedtime.

Your Answer. c
Correct Answer. d

Copyright 2014 Delhi Academy of Medical Sciences, All Rights Reserved. 52/84
(125). Cardiotoxicity is a well-known side effect of several cytotoxic drugs. Which of the following cytotoxic drugs is most likely to lead to long
term morbidity due to cardiotoxicity ?

a. Doxorubicin

b. 5-fluorouracil

c. Taxoids

d. Cyclopentenyl cytosine

Solution. Ans 125: (a) Doxorubicin


Ref - Read the text below
Sol:
* Cardiotoxicity is a well-known side effect of several cytotoxic drugs, especially of the anthracyclines and can lead to long term
morbidity.
* The mechanism of anthracycline induced cardiotoxicity seems to involve the formation of free radicals leading to oxidative stress.
* This may cause apoptosis of cardiac cells or immunologic reactions.

Your Answer. Not Attempted


Correct Answer. a

(126). Phenoxybenzamine is best described as :

a. Alpha-receptor antagonist

b. Anticholinergic

c. Muscarinic agonist

d. Beta blocker

Solution. Ans 126: (a) Alpha-receptor antagonist


Ref Read the text below
Sol:
* An example of a selective alpha-receptor antagonist is phenoxybenzamine
* Phenoxybenzamine belongs to a class of medicines called the alpha adrenergic blockers. It works by blocking alpha receptors in certain
parts of the body. Alpha receptors are present in the muscle that lines the walls of blood vessels.
* When the receptors are blocked by phenoxybenzamine, the muscle relaxes and the blood vessels widen. This widening of the blood
vessels results in a lowering of blood pressure. This medicine can be used to treat high blood pressure.
* This medicine is used in the short term treatment of severe hypertensive episodes associated with phaeochromocytoma

Your Answer. a
Correct Answer. a

(127). The most useful medication that may be used in absence seizures

a. Phenytoin

b. Primidone

c. Carbamazepine

d. Valproic acid

Solution. Ans 127: (d) Valproic acid


Ref Read the text below
Sol:
Valproic acid is a drug of choice for:
* Generalized epilepsy of unknown cause that produces more than one type of seizure.
* Absence seizures.Ethosuximide is the other drug of choice. If a person has absence seizures as well as other types of seizures, however,
valproic acid usually works better because it can control several types of seizures.
* Myoclonic seizures, such as those caused by juvenile myoclonic epilepsy.
* Clonazepam may be used when valproic acid does not work. Valproic acid is also considered a first-line drug for treating partial
seizures.

Your Answer. d
Correct Answer. d

Copyright 2014 Delhi Academy of Medical Sciences, All Rights Reserved. 53/84
(128). Contra-indicators to -blockers include all of the following except

a. Raynaud's disease

b. Wolff-Parkinson-White syndrome

c. Chronic obstructive pulmonary disease

d. Intermittent claudication

Solution. Ans 128 : (b) Wolff-Parkinson-White syndrome


Ref:Read the text below
Sol:
The three classic groups of patients where -blockers are contra-indicated are
*Peripheral vascular disease and Raynaud's
* Asthma and COPD
* Heart failure
* As well as blockade of -receptors, some of these drugs may have additional features such as cardioselectivity, partial agonist activity
and membrane-stabilising activity.
* Cardioselective -blockers such as acebutalol, atenolol, bisoprolol, metoprolol and esmolol block the 1-receptors to a greater degree
than the 2-receptors.
* Some of the -blocking drugs e.g. acebutalol and oxprenolol have partial agonist activity, sometimes described as intrinsic
sympathomimetic activity (ISA). It has been suggested that drugs with this property have a lower incidence of the undesirable side-
effects such as bradycardia, heart failure, cold extremities and bronchospasm.
* Propranolol has membrane-stabilising activity, potent local anaesthetic properties and demonstrates class II anti-arrhythmic properties.

Your Answer. d
Correct Answer. b

(129). Drug combinations that have no significant clinical interaction include

a. Trimethoprim an phenytoin

b. Etomidate and hydrocortisone

c. Rifampicin and propofol

d. Ciprofloxacin and aminophylline

Solution. Ans 129 : (b) Etomidate and hydrocortisone


Ref:Read the text below
Sol:
* Trimethoprim and phenytoin interfere with folate metabolism causing megaloblastic anaemia.
* Rifampicin is a liver enzyme inducer which will increase the metabolism of drugs metabolised by the liver.
* Ciprofloxacin and erythromycin are liver enzyme inhibitors which will decrease the metabolism of drugs metabolised by the liver.

Your Answer. b
Correct Answer. b

Copyright 2014 Delhi Academy of Medical Sciences, All Rights Reserved. 54/84
(130). Tramadol

a. Is a controlled drug

b. Has affinity for binding at the mu opioid receptor comparable to that of morphine

c. Acts predominantly by inhibiting the reuptake of noradrenaline and serotonin (5-HT)

d. May be used concurrently with a MAOI

Solution. Ans 130 : (c) Acts predominantly by inhibiting the reuptake of noradrenaline and serotonin (5-HT)
Ref:Read the text below
Sol:
* Tramadol is not a controlled drug. It has a weak affinity for opioid receptors.
* The affinity for binding at the mu receptor is 2.1 M compared with 0.0003 M for morphine.
* Its main mode of action is the modulation of central monoaminergic pathways.
* As with pethidine (and possibly other opioids), the use of tramadol with MAOI is contraindicated. Both drugs delay the removal of 5-HT
from its site of action.
* The interaction may presents clinically as sudden agitation, unmanageable behaviour, headaches, hypertension or hypotension, rigidity,
hyperpyrexia, convulsions and coma

Your Answer. b
Correct Answer. c

(131). The metabolism of which of the following does not follow zero-order kinetics in clinical doses

a. Phenytoin

b. Aspirin

c. Ethanol

d. Propranolol

Solution. Ans 131 : (d) Propranolol


Ref:Read the text below
Sol:
The metabolism of the following observe zero-order kinetics
* Phenytoin
* Aspirin
* Ethanol
* Paracetamol (in toxic doses)
* Thiopentone (in toxic doses)

Your Answer. d
Correct Answer. d

Copyright 2014 Delhi Academy of Medical Sciences, All Rights Reserved. 55/84
(132). The following are examples of hepatic enzyme inducer

a. Cimetidine

b. Erythromycin

c. Griseofulvin

d. Amiodarone

Solution. Ans 132 : (c) Griseofulvin


Ref:Read the text below
Sol:

Your Answer. a
Correct Answer. c

(133). Hepatic first-pass metabolism

a. Is avoided by giving the drug transdermally

b. Is avoided by giving the drug sublingually

c. Is avoided by giving the drug rectally

d. Is seen when a drug has a high hepatic extraction ratio

Solution. Ans 133 : (c) Is avoided by giving the drug rectally


Ref:Read the text below
Sol:
* Transdermal, sublingual and parenteral (IV, IM, SC) routes avoid the hepatic portal circulation and hence the hepatic first pass
metabolism.
* A variable proportion of the drug given rectally will be absorbed into the portal circulation, therefore first pass metabolism is not totally
avoided.
* Drugs with high hepatic extraction ratios undergo substantial first pass metabolism

Your Answer. b
Correct Answer. c

Copyright 2014 Delhi Academy of Medical Sciences, All Rights Reserved. 56/84
(134). Select incorrect statement regarding First-order kinetics

a. A constant proportion of drug metabolised in a given time period

b. The absolute amount eliminated is greatest when plasma concentration is greatest

c. The rate of elimination and elimination half-life is constant, irrespective of plasma concentration

d. The enzyme responsible for the reaction is saturated

Solution. Ans 134 : (d) The enzyme responsible for the reaction is saturated
Ref:Read the text below
Sol:
Zero-order kinetics are observed when the enzyme responsible for the reaction is saturated and when the reaction is represented by a
linear relationship. Summary of the differences between zero-order and first-order kinetics: Zero-order:
* Absolute amount eliminated is the same, regardless of plasma concentration;
* Rate of elimination varies with the plasma concentration;
* Constant amount of drug metabolised per unit time;
* Reflects saturation of enzyme: occurs when plasma concentration exceeds capacity of enzyme.
First-order:
* Constant proportion of drug metabolised in a given time period;
* Absolute amount eliminated is greatest when plasma concentration is greatest;
* Rate of elimination and elimination half-life are constant, irrespective of concentration.

Your Answer. b
Correct Answer. d

(135). The efficacy of a drug

a. Is greater for drug A if A is effective in a dose of 100 g than for drug B if B is effective in a dose of 100 mg

b. Is a measure of its therapeutic index

c. Is a measure of the amount of a drug required to produce a given effect

d. Describes the ability of a drug to produce its therapeutic effect

Solution. Ans 135 : (d) Describes the ability of a drug to produce its therapeutic effect
Ref:Read the text below
Sol:
* The dose of a drug required to produce a given effect decribes its potency, not its efficacy.
* In the example described, drug A is more potent than drug B.
* The therapeutic index of a drug is a measure of its safety (ED50/LD50).
* Efficacy, however, is a measure of the maximal effect of an agonist.

Your Answer. c
Correct Answer. d

Copyright 2014 Delhi Academy of Medical Sciences, All Rights Reserved. 57/84
(136). A new drug is being studied to find the most appropriate dose in a dose response study. Small doses of the drug lead to a linear increase
in serum drug concentration. At higher doses there is an exponential rise in serum drug concentration. Which of the following best
describes the pharmacokinetic properties of this new drug?

a. First pass effect

b. Long plasma half life

c. Saturation kinetics

d. Zero order kinetics

Solution. Ans 136 : (c) Saturation kinetics


Ref:Read the text below
Sol:
* The description of the kinetics of this new drug show that with small doses there is a linear response (first order kinetics) to dosing but
this becomes saturated and the serum concentration of the drug rises sharply (zero order kinetics).
* This response is typical of drugs such as phenytoin (saturates liver metabolism).

Your Answer. d
Correct Answer. c

(137). Which of the following drugs would exacerbate Parkinson's disease?

a. Amantadine

b. Bromocriptine

c. Chlorpromazine

d. Levodopa/Carbidopa

Solution. Ans-137: (c) Chlorpromazine


Reference Read the text below
Sol:
* Chlorpromazine (Thorazine) is a dopamine antagonist and will worsen the symptoms of Parkinson's disease.

Your Answer. c
Correct Answer. c

Copyright 2014 Delhi Academy of Medical Sciences, All Rights Reserved. 58/84
(138). The latest and most effective therapy for AIDS patients includes azidothymidine (AZT), dideoxyinosine (DDI), and saquinavir or similar
agents. Use of these three drugs would inhibit which of the following viral processes?

a. RNase, DNase

b. p24 antibody expression

c. All membrane synthesis

d. Reverse transcriptase, protease

Solution. Ans-138: (d) Reverse transcriptase, protease


Reference Read the text below
Sol:
* The advent of triple therapy or a therapeutic "cocktail" has had a marked effect on AIDS patients.
* The combination of drugs work together as reverse transcriptive inhibitors and a protease inhibitor. Patients improve rapidly, their CD4
lymphocyte counts increase, and their HIV viral load is drastically reduced, often to <50 copies per ml.
* On the other hand, an untreated HIV-positive patient with a low CD4 and a high viral load (a) is at increased risk of opportunistic
infection and (b) has a much greater chance of developing AIDS than if the viral load was <50,000. The patient is infectious and his HIV
antibody screening test will be positive.
* The high viral load, however, is not a predictor of response to therapy. Many patients with high viral loads do very well on triple
therapy, although resistance to one or more of the agents may subsequently occur. A low CD4 count does not predict progression to
AIDS but does indicate increased chance of opportunistic infection such as those listed.
* Kaposis sarcoma, which has been linked to herpesvirus type 8, pneumocystis, and mycobacterial disease are three of the most
prevalent opportunistic infections. While HIV-positive patients contract pneumococcal pneumonia, they are probably at no more risk than
the general population, as protection against pneumococcal disease is linked to the presence of anticapsular antibody.

Your Answer. d
Correct Answer. d

(139). The first drug to be effective against AIDS, including the reduction of maternal-to-child AIDS transmission by 30%, was AIDS drug
azidothymidine (AZT). Which of the following describes its mechanism of action?

a. It inhibits RNA synthesis

b. It inhibits viral DNA polymerase

c. It stimulates DNA provirus production

d. It inhibits viral reverse transcriptase

Solution. Ans-139: (d) It inhibits viral reverse transcriptase


Reference Read the text below
Sol:
* The AIDS treatment drug azidothymidine (AZT) exerts its effect by inhibiting viral reverse transcriptase.
* Thus, it prevents replication of the human immunodeficiency virus. Reverse transcriptase is an RNA-directed DNA polymerase.
* The RNA of retroviruses utilizes reverse transcriptase to synthesize DNA provirus, which in turn synthesizes new viral RNA.
* AZT inhibits DNA provirus production, but does not directly inhibit synthesis of new viral RNA.

Your Answer. d
Correct Answer. d

Copyright 2014 Delhi Academy of Medical Sciences, All Rights Reserved. 59/84
(140). The first H2 receptor blocker was synthesized in 1972 by

a. Asch and Child

b. Sir James Black

c. Gaddum and Picarelli

d. Vane

Solution. Ans-140: (b) Sir James Black


Ref.: Read the text below
Sol :
Sir James Black. He synthesized the first H2 blocker burimamide; and subsequently cimetidine.
* Asch and Child classified histamine receptors into H1 and H2.
* Gaddum and Picarelli classified serotonin receptors into D type and M type.
* Vane observed that aspirin acts by inhibiting PG synthesis.

Your Answer. c
Correct Answer. b

(141). The antiretroviral drug that can bind to both CCR2 and CCR5 is -

a. Cenicriviroc

b. Vicriviroc

c. Ipalizumab

d. Bevirimat

Solution. Ans-141: (a) Cenicriviroc


Ref.: Read the text below
Sol :
Cenicriviroc
* Cenicriviroc binds to both CCR2 and CCR5. It is a fusion / entry inhibitor. CCR2 binding may contribute to anti-inflammatory effect.
* Vicriviroc binds to CCR5. It is a fusion / entry inhibitor. Not found to be efficacious in clinical trials, hence further trials were
discontinued.
* Ipalizumab is a monoclonal antibody against CD4. It is a fusion / entry inhibitor
* Bevirimat is a maturation inhibitor. It binds to gag protein

Your Answer. a
Correct Answer. a

(142). Which finding on electron microscopy indicates irreversible cell injury


Bleomycin affects which of the following

a. a. Type 1 pneumocytes

b. b. Type 2 pneumocytes

c. c. Both Type 1 and Type 2 pneumocytes

d. d. Pulmonary endothelial cells

Solution. Ans-142: (a) Type 1 pneumocytes


Ref.: Read the text below
Sol : Type 1 pneumocytes. Bleomycin causes damage to Type 1 pneumocytes with compensatory hyperplasia of Type 2 pneumocytes.

Your Answer. a
Correct Answer. a

Copyright 2014 Delhi Academy of Medical Sciences, All Rights Reserved. 60/84
(143). Which of the following is approved for the treatment of cystic fibrosis with G551D mutation?

a. Biostrophin

b. Peginesatide

c. Lumacaftor

d. Ivacaftor

Solution. Ans-143: (d) Ivacaftor


Ref.: Read the text below
Sol : Ivacaftor
* Biostrophin is a vehicle for gene therapy in Duchenne and Beckers muscular dystrophy
* Peginesatide is a erythropoietin analogue for anemia of chronic kidney disease
* Lumacaftor is a CFTR potentiator for cystic fibrosis with F508 mutation
* Ivacaftor is a CFTR potentiator for cystic fibrosis with G551D mutation

Your Answer. b
Correct Answer. d

(144). The drug of choice for an acute exacerbation of ulcerative colitis is

a. Sulfasalazine

b. Methotrexate

c. Infliximab

d. Corticosteroids

Solution. Ans-144: (d) Corticosteroids


Ref.: Read the text below
Sol : Corticosteroids
* DOC for acute exacerbation of IBD Steroids
* DOC for maintenance treatment of IBD Sulfasalazine

Your Answer. a
Correct Answer. d

(145). All of the following are anti-craving agents for alcohol withdrawl except

a. Fluoxetine

b. Nitrafezole

c. Naltrexone

d. Acamprosate

Solution. Ans-145: (b) Nitrafezole


Ref.: Read the text below
Sol :
Nitrafezole. Disulfiram, Metronidazole and Nitrafezole are deterrents.

Your Answer. a
Correct Answer. b

Copyright 2014 Delhi Academy of Medical Sciences, All Rights Reserved. 61/84
(146). Which of the following drugs for moderate-to-severe psoriasis inhibits both IL-12 and IL23?

a. Alefacept

b. Efalizumab

c. Ustekinumab

d. Abatacept

Solution. Ans-146: (c) Ustekinumab


Ref.: Read the text below
Sol : Ustekinumab
* Alefacept is anti CD-2 for psoriasis
* Efalizumab is anti CD-11a for psoriasis
* Ustekinumab targets the p40 subunit of both IL-12 and IL-23 in psoriasis
* Abatacept is anti CD-80 and CD-86 for severe RA

Your Answer. d
Correct Answer. c

(147). Which of the following enzymes can polymerize deoxyribonucleotides into DNA?

a. DNA ligase

b. DNA gyrase

c. RNA polymerase III

d. Reverse transcriptase

Solution. Ans-147: (d) Reverse transcriptase


Reference Read the text below
Sol:
* Reverse transcriptase is an RNA-dependent DNA polymerase that can synthesize first a single strand and then a double-stranded DNA
from a single-strand RNA template.
* It was originally found in animal retroviruses. Primase is a DNA-dependent RNA polymerase enzyme that synthesizes an RNA molecule
10 to 200 nucleotides in length that initiates or "primes" DNA synthesis. DNA ligase joins DNA fragments and DNA gyrase winds or
unwinds DNA.
* Transfer RNA, 5SRNA, and other small RNAs are synthesized by RNA polymerase III (RNA polymerase I synthesizes ribosomal RNA
and RNA polymerase II synthesizes messenger RNA).

Your Answer. c
Correct Answer. d

(148). Hypotension, bradycardia, respiratory depression, and muscle weakness,all unresponsive to atropine and neostigmine, would most likely
be due to

a. Diazoxide

b. Isofluorphate

c. Tubocurarine

d. Nicotine

Solution. Ans 148: (d) Nicotine.


Reference Read the text below
Sol:
* Nicotine is a depolarizing ganglionic blocking agent that initially stimulates and then blocks nicotinic muscular (NM) (skeletal muscle)
and nicotinic neural (NN) (parasympathetic ganglia) cholinergic receptors.
* Blockade of the sympathetic division of the autonomic nervous system (ANS) results in arteriolar vasodilation, bradycardia, and
hypotension.
* Blockade at the neuromuscular junction leads to muscle weakness and respiratory depression caused by interference with the function
of the diaphragm and intercostal muscles.

Your Answer. c
Correct Answer. d

Copyright 2014 Delhi Academy of Medical Sciences, All Rights Reserved. 62/84
(149). A predictably dangerous side effect of nadolol that constitutes a contraindication to its clinical use in susceptible patients is the induction
of

a. Hypertension

b. Cardiac arrhythmia

c. Asthmatic attacks

d. Respiratory depression

Solution. Ans 149 : (c) Asthmatic attacks


Ref Read the text below
Sol:
* The chief danger of therapy with -adrenergic blocking agents, such as nadolol and propranolol, is associated with the blockade itself.
-adrenergic blockade results in an increase in airway resistance that can be fatal in asthmatic patients.
* Hypersensitivity reactions such as rash, fever, and purpura are rare and necessitate discontinuation of therapy.

Your Answer. b
Correct Answer. c

(150). The cholinomimetic drug that is useful for treating postoperative abdominal distention and gastric atony is

a. Acetylcholine (ACh)

b. Methacholine

c. Carbachol

d. Bethanechol

Solution. Ans 150 : (d) Bethanechol


Ref Read the text below
Sol:
* Of the four choline esters (ACh, methacholine, carbachol, and bethanechol), the latter two drugs have the greatest agonistic activity on
muscarinic receptors of the GI tract and urinary bladder.
* Bethanechol is used orally or by subcutaneous injection as a stimulant of the smooth muscles of the GI tract (for cases of postoperative
abdominal distention, gastric atony, and retention or gastroparesis) and the urinary bladder (for nonobstructive postoperative and
postpartum urinary retention).
* Carbachol is not used for these purposes due to significant activity at nicotinic receptors at autonomic ganglia; the drug is useful as a
miotic for treating glaucoma and in certain types of ocular surgery.
* Acetylcholine is occasionally used topically during cataract surgery; metacholine is used by inhalation for the diagnosis of bronchial
hyperreactivity in patients who do not have clinically apparent asthma.

Your Answer. b
Correct Answer. d

Copyright 2014 Delhi Academy of Medical Sciences, All Rights Reserved. 63/84
(151). Which of the following cardiovascular agents is classified chemically as a glycoside?

a. Nifedipine

b. Digoxin

c. Flecainide

d. Cholestyramin

Solution. Ans-151: (b) Digoxin


Reference Read the text below
Sol:
* Most glycosides are natural products obtained from plant material. Although there are very few medicinal agents that are glycosides,
the group known as the cardiac glycosides are extremely important and they are widely used for treating congestive heart failure.
* Digoxin is a cardiac glycoside obtained from Digitalis lanata.
* Other cardiac glycosides include digitoxin, which is obtained from Digitalis purpurea, and ouabain, which is obtained from
Strophanthus gratus.

Your Answer. b
Correct Answer. b

(152). Drug bound to alpha 1 acid glycoprotein are all except :

a. Lignocaine

b. Tolbutamide

c. Quinidine

d. Propranolol

Solution. Ans-152: (b) Tolbutamide


Ref.: KDT 6th p. 12

Your Answer. Not Attempted


Correct Answer. b

Copyright 2014 Delhi Academy of Medical Sciences, All Rights Reserved. 64/84
(153). A patient is on propanolol. Adverse affects of beta blocker are all except

a. Seizures

b. Rebound tachycardia

c. Hypotension

d. Poor response to hypoglycemia

Solution. Ans-153: (b) Rebound tachycardia


Ref.: Martindale 6th ed. P. 1226
Sol :
* Among the most serious adverse effects are heart failure, heart block, and bronchospasm.
* Troublesome subjective effects include fatigue and coldness of the extremities.
* Cardiovascular effects include bradycardia and hypotension; heart failure or heart block may be precipitated or worsened in patients
with underlying cardiac disorders.
* CNS effects include headache, depression, dizziness, hallucinations, confusion, amnesia, and sleep disturbances including nightmares.
* Coma and convulsions have been reported after beta-blocker overdosage.
* Effects on carbohydrate metabolism. The sympathetic nervous system is involved in the control of carbohydrate metabolism and beta
blockers can interfere with carbohydrate and insulin regulation; both hypoglycaemia and hyperglycaemia have been reported in patients
with no history of diabetes, as well as in patients with types 1 or 2 diabetes mellitus.
* Beta blockers cause hypoglycaemia in non-diabetics, possibly by increasing peripheral glucose uptake through increased insulin
sensitivity.
* Beta blockers may mask the adrenaline mediated symptoms of hypoglycaemia such as tachycardia and tremor, and non-
cardioselective beta blockers may delay recovery in patients given glucose for hypoglycaemia.

Your Answer. a
Correct Answer. b

(154). Which drug acts by inhibiting protein synthesis?

a. Ciprofloxacin

b. Penicillin

c. Erythromycin

d. Nalidixic acid

Solution. Ans-154: (c) Erythromycin


Ref.: Katzung -805
Sol : Drugs acting on 50 S
* chloramphenicol, erythromycin, lincosamides, linezolide, streptogramin.

Your Answer. d
Correct Answer. c

Copyright 2014 Delhi Academy of Medical Sciences, All Rights Reserved. 65/84
(155). Which of the following drug has lipemia clearing action?

a. Warfarin

b. Heparin

c. Epsilon amino caproic acid

d. Streptokinase

Solution. Ans-155 : (b) Heparin


Ref.: Read the text below
Sol :
* Heparin releases lipoprotein lipase from the vessel walls and tissues which hydrolyses triglycerides of chylomicroms and VLDL to free
fatty acids.
* This action is seen with lower doses of heparin than what isrequired for anticoagulation
* This effect is only seen invivo and not invitro.
* Heparin, in addition to anticoagulation also inhibits platelet aggregation and prolongs bleeding time.

Your Answer. d
Correct Answer. b

(156). Most specific drug for the treatment of peptic ulcer caused due to chronic use of NSAIDs is

a. Rabeprazole

b. Loxatidine

c. Misoprostol

d. Esomeprazole

Solution. Ans-156: (c) Misoprostol


Ref: KDTs - 634
Sol:
* PGE1 analog, misoprostol, is the most specific drug for the treatment of NSAID induced peptic ulcer disease whereas proton pump
inhibitors are the agents of choice for all types of peptic ulcer disease including NSAID-induced.
* Misoprostol is approved for use in the prevention of NSAID-induced gastric ulcers. It acts upon gastric parietal cells, inhibiting the
secretion of gastric acid via G-protein coupled receptor-mediated inhibition of adenylate cyclase, which leads to decreased intracellular
cyclic AMP levels and decreased proton pump activity at the apical surface of the parietal cell.

Your Answer. c
Correct Answer. c

(157). The drug of choice for rapid correction of PSVT in known asthmatic is

a. Adenosine

b. Esmolol

c. Neostigmine

d. Verapamil

Solution. Ans-157: (d) Verapamil


Ref: KDT -484
Sol:
* Adenosine is the DOC for acute termination of PSVT. Esmolol and verapamil are alternative 2nd choice drugs. However adenosine may
precipitate bronchospasm in asthmatics, so not preferred in asthmatics.
* Beta-blockers should be avoided in asthmatics, however if absolutely necessary, cardioselective blockers (e.g. esmolol) should be used.
* Further in asthmatic patients, verapamil (as well as nifedipine) given by inhalation significantly inhibits the bronchocostriction induced
by variety of stimuli.

Your Answer. d
Correct Answer. d

Copyright 2014 Delhi Academy of Medical Sciences, All Rights Reserved. 66/84
(158). Which of the following is wrongly matched combination of anti-arrhythmic drugs and their class

a. Mexiletine-IB

b. Verapamil-IV

c. Amiodarone-III

d. Lignocaine-IA

Solution. Ans-158: (d) Lignocaine IA


Ref: KDTs - 475

Your Answer. a
Correct Answer. d

Copyright 2014 Delhi Academy of Medical Sciences, All Rights Reserved. 67/84
(159). Which of the following has the maximum half life?

a. Adenosine

b. Amiodarone

c. Esmolol

d. Lidocaine

Solution. Ans-159: (b) Amiodarone


Ref: Read the text below.
Sol:
* Amiodarone is an antiarrhythmic agent (medication used for irregular heart beat) used for various types of tachyarrhythmias (fast
forms of irregular heart beat), both ventricular and supraventricular (atrial) arrhythmias.
* Excretion is primarily hepatic and biliary with almost no elimination via the renal route and it is not dialyzable.
* Elimination half-life average of 58 days (ranging from 25100 days
* There is 10-50% transfer of amiodarone and DEA in the placenta as well as presence in breast milk.
* Accumulation of amiodarone and DEA occurs in adipose tissue and highly perfused organs (i.e. liver, lungs) therefore, if an individual
was taking amiodarone on a chronic basis, if it is stopped it will remain in the system for weeks to months

Your Answer. b
Correct Answer. b

(160). The 5 reductase inhibitor that has been found to be effective both in benign prostatic hypertrophy and male pattern baldness is

a. Flutamide

b. Finasteride

c. Prazosin

d. Minoxidil

Solution. Ans-160: (b) Finasteride


Ref: KDTs 6th Pg. 294, 295
Sol:
* Finasteride is used for the treatment of benign prostatic hyperplasia (BPH) (also known as enlarged prostate) at a dose of 5 mg once a
day. It may take six months or more to see the full effects of finasteride. If the drug is discontinued, any therapeutic benefits will be
reversed. Finasteride may improve the symptoms associated with BPH such as difficulty urinating, getting up during the night to urinate,
hesitation at the start of urination, and decreased urinary flow.
* In a 5-year study of men with mild to moderate hair loss, 2 out of 3 of the men who took 1 mg of finasteride daily regrew some hair, as
measured by hair counts

Your Answer. b
Correct Answer. b

Copyright 2014 Delhi Academy of Medical Sciences, All Rights Reserved. 68/84
(161). Which of the following drugs is preferred for long term treatment of severe anxiety disorder with intermittent panic attacks ?

a. Phenothiazine

b. Azapirone

c. blocker

d. Selective serotonin reuptake inhibitor

Solution. Ans-161: (d) Selective serotonin reuptake inhibitor Ref: Harrison 16th Ed. Pg. 2548 Sol: Appropriate medication is highly
effective for panic disorder. Although there is little evidence that pharmacological interventions can directly alter phobias, few studies
have been performed, and medication treatment of panic makes phobia treatment far easier. Medications can include:
Antidepressants (SSRIs, MAOIs, tricyclic antidepressants): these are taken regularly every day, and alter neurotransmitter
configurations which in turn can help to block symptoms. Although these medications are described as "antidepressants", nearly all of
them especially the tricyclic antidepressants have anti-anxiety properties, in part, due to their sedative effects. SSRIs have been
known to exacerbate symptoms in panic disorder patients, especially in the beginning of treatment and have even provoked panic attacks
in otherwise healthy individuals. SSRIs are also known to produce withdrawal symptoms which include rebound anxiety and panic
attacks. Comorbid depression has been cited as imparting the worst course, leading to chronic, disabling illness.
Anti-anxiety drugs (benzodiazepines): Use of benzodiazepines for panic disorder is controversial with opinion differing in the medical
literature. The American Psychiatric Association states that benzodiazepines can be effective for the treatment of panic disorder and
recommends that the choice of whether to use benzodiazepines, antidepressants with antipanic properties or psychotherapy should be
based on the individual patients history and characteristics. They reported that in their view there is insufficient evidence to recommend
one treatment over another for panic disorder. The APA noted that while benzodiazepines have the advantage of a rapid onset of action,
that this is offset by the risk of developing a benzodiazepine dependence.
The National Institute of Clinical Excellence came to a different conclusion, they pointed out the problems of using uncontrolled clinical
trials to assess the effectiveness of pharmacotherapy and based on placebo controlled research they concluded that benzodiazepines
were not effective in the long-term for panic disorder and recommended that benzodiazepines not be used for longer than 4 weeks for
panic disorder. Instead NICE clinical guidelines recommend alternative pharmacotherapeutic or psychotherapeutic interventions.
Other experts believe that benzodiazepines are best avoided due to the risks of the development of tolerance and physical dependence.
The World Federation of Societies of Biological Psychiatry, say that benzodiazepines should not be used as a first line treatment option
but are an option for treatment resistant cases of panic disorder.
Despite increasing focus on the use of antidepressants and other agents for the treatment of anxiety as recommended best practice,
benzodiazepines have remained a commonly used medication for panic disorder

Your Answer. d
Correct Answer. d

Copyright 2014 Delhi Academy of Medical Sciences, All Rights Reserved. 69/84
(162). A patient of endogenous depression was prescribed imipramine. After what duration of time interval is the therapeutic effect of
imipramine likely to manifest?

a. Three days

b. One week

c. Three weeks

d. Three months

Solution. Ans-162: (c) Three weeks Ref: KDT--441 Sol: The mechanism of action of imipramine HCl is not definitely known. However, it
does not act primarily by stimulation of the central nervous system. The clinical effect is hypothesized as being due to potentiation of
adrenergic synapses by blocking uptake of norepinephrine at nerve endings. The mode of action of the drug in controlling childhood
enuresis is thought to be apart from its antidepressant effect.
INDICATIONS AND USAGE Depression For the relief of symptoms of depression. Endogenous depression is more likely to be alleviated
than other depressive states. One (1) to 3 weeks of treatment may be needed before optimal therapeutic effects are evident. Childhood
Enuresis
May be useful as temporary adjunctive therapy in reducing enuresis in children aged 6 years and older, after possible organic causes
have been excluded by appropriate tests.
In patients having daytime symptoms of frequency and urgency, examination should include voiding cystourethrography and cystoscopy,
as necessary. The effectiveness of treatment may decrease with continued drug administration.
CONTRAINDICATIONS
The concomitant use of monoamine oxidase inhibiting compounds is contraindicated.
Hyperpyretic crises or severe convulsive seizures may occur in patients receiving such combinations.
The potentiation of adverse effects can be serious, or even fatal. When it is desired to substitute imipramine HCl in patients receiving a
monoamine oxidase inhibitor, as long an interval should elapse as the clinical situation will allow, with a minimum of 14 days. Initial
dosage should be low and increases should be gradual and cautiously prescribed.
The drug is contraindicated during the acute recovery period after a myocardial infarction.
Patients with a known hypersensitivity to this compound should not be given the drug. The possibility of cross-sensitivity to other
dibenzazepine compounds should be kept in mind.

Your Answer. c
Correct Answer. c

(163). Drug of choice for methicillin resistant staph aureus is

a. Vancomycin

b. Ciprofloxacin

c. Carbenicillin

d. Nystatin

Solution. Ans-163: (a) Vancomycin Ref: Read the text below. Sol:
Both CA-MRSA and HA-MRSA are resistant to traditional anti-staphylococcal beta-lactam antibiotics, such as cephalexin. CA-MRSA has
a greater spectrum of antimicrobial susceptibility, including to sulfa drugs (like co-trimoxazole/trimethoprim-sulfamethoxazole),
tetracyclines (like doxycycline and minocycline) and clindamycin, but the drug of choice for treating CA-MRSA has is now believed to be
Vancomycin, according to a Henry Ford Hospital Study
Vancomycin and teicoplanin are glycopeptide antibiotics used to treat MRSA infections.
Teicoplanin is a structural congener of vancomycin that has a similar activity spectrum but a longer half-life.
Because the oral absorption of vancomycin and Teicoplanin is very low, these agents must be administered intravenously to control
systemic infections.
Drugs are administered via a Peripherally inserted central catheter, or a Picc Line, which is inserted by radiologists, doctors, physician
assistants (in the U.S.), radiologist assistants (in the U.S.), or specially trained certified registered nurses. Treatment of MRSA infection
with vancomycin can be complicated, due to its inconvenient route of administration. Moreover, many clinicians believe that the efficacy
of vancomycin against MRSA is inferior to that of anti-staphylococcal beta-lactam antibiotics against MSSA.
Several newly discovered strains of MRSA show antibiotic resistance even to vancomycin and teicoplanin. These new evolutions of the
MRSA bacterium have been dubbed Vancomycin intermediate-resistant Staphylococcus aureus (VISA). Linezolid,
quinupristin/dalfopristin(synercid), daptomycin, and tigecycline are used to treat more severe infections that do not respond to
glycopeptides such as vancomycin.

Your Answer. a
Correct Answer. a

Copyright 2014 Delhi Academy of Medical Sciences, All Rights Reserved. 70/84
(164). Which of the following drugs is most likely to cause loss of equilibrium and auditory damage?

a. Amikacin

b. Ethambutol

c. Isoniazid

d. Rifabutin

Solution. Ans-164: (a) Amikacin Ref: KDT--721 Sol: Amikacin


Nephrotoxicity
Ototoxicity Hearing loss
Vestibular damage
Neuromuscular block
Side effects of amikacin are similar to other aminoglycosides. Kidney damage and hearing loss are the most important effects.
Because of this potential, blood levels of the drug and markers of kidney function (creatinine) may be monitored.
Moreover, doses are adjusted specifically based upon serum Creatinine clearance in clinical settings

Your Answer. b
Correct Answer. a

(165). Which of the following anticancer drugs has highest emetogenic potential?

a. Vincristine

b. Chlorambucil

c. 6-Mercaptopurine

d. Cisplatin

Solution. Ans-165: (d) Cisplatin Ref: Read the text below Sol: Cisplatin has a number of side-effects that can limit its use:
Nephrotoxicity (kidney damage) is a major concern. The dose is reduced when the patient's creatinine clearance (a measure of renal
function) is reduced. Adequate hydration and diuresis is used to prevent renal damage. The nephrotoxicity of platinum-class drugs seems
to be related to reactive oxygen species and in animal models can be ameliorated by free radical scavenging agents (e.g., amifostine).
Nephrotoxicity is a dose-limiting.
Neurotoxicity (nerve damage) can be anticipated by performing nerve conduction studies before and after treatment.
Nausea and vomiting: cisplatin is one of the most emetogenic chemotherapy agents, but this symptom is managed with prophylactic
antiemetics (ondansetron, granisetron, etc.) in combination with corticosteroids. Aprepitant combined with ondansetron and
dexamethasone has been shown to be better for highly emetogenic chemotherapy than just ondansetron and dexamethasone.
Ototoxicity (hearing loss): unfortunately there is at present no effective treatment to prevent this side effect, which may be severe.
Audiometric analysis may be necessary to assess the severity of ototoxicity. Other drugs (such as the aminoglycoside antibiotic class)
may also cause ototoxicity, and the administration of this class of antibiotics in patients receiving cisplatin is generally avoided. The
ototoxicity of both the aminoglycosides and cisplatin may be related to their ability to bind to melanin in the stria vascularis of the inner
ear or the generation of reactive oxygen species.
Electrolyte disturbance: Cisplatin can cause hypomagnesaemia, hypokalaemia and hypocalcaemia. The hypocalcaemia seems to occur in
those with low serum magnesium secondary to cisplatin, so it is not primarily due to the Cisplatin.

Your Answer. d
Correct Answer. d

Copyright 2014 Delhi Academy of Medical Sciences, All Rights Reserved. 71/84
(166). The penicillin G preparation with the longest duration of action is

a. Benzathine penicillin

b. Sodium penicillin

c. Potassium penicillin

d. Procaine penicillin

Solution. Ans-166: (a) Benzathine penicillin Ref: Read the text below Sol:
Benzathine benzylpenicillin (rINN) is a form of penicillin also known as benzathine penicillin.
It is slowly absorbed into the circulation, after intramuscular injection, and hydrolysed to benzylpenicillin in vivo.
It is the drug-of-choice when prolonged low concentrations of benzylpenicillin are required and appropriate, allowing prolonged
antibiotic action over 24 weeks after a single IM dose

Your Answer. a
Correct Answer. a

(167). Which fluoroquinolone is highly active against Mycobacterium leprae and is being used in alternative multidrug therapy regimens

a. Norfloxacin

b. Ofloxacin

c. Ciprofloxacin

d. Lomefloxacin

Solution. Ans-167 (b) Ofloxacin Ref: KDTs 6th Ed. Pg. 756 Sol: Single lesion single dose treatment of leprosy utilizes ROM therapy.
R : rifampicin
O : Ofloxacin
M : Minocycline

Your Answer. b
Correct Answer. b

(168). The fastest acting schizontocidal drug among the following is

a. Artemether

b. Mefloquine

c. Chloroquine

d. Proguanil

Solution. Ans-168: (a) Artemether Ref: KDT-792 Sol:


Artemisinin derivatives like dihydroartemisinin, arteether and artemether etc. are fastest acting antimalarial drugs.
Ans 169: (d) Less neurotoxic then cyclophosphamide Ref Read the text below Sol
Ifosfamide is an oxazaphosphorine, similar to cyclophosphamide.
Cyclophosphamide has two chloroethyl groups on the exocyclic nitrogen atom, whereas one of the two-chloroethyl groups of ifosfamide
is on the cyclic phosphoamide nitrogen of the oxazaphosphorine ring.
Ifosfamide is activated in the liver by CYP3A4. The activation of ifosfamide proceeds more slowly, with greater production of
dechlorinated metabolites and chloroacetaldehyde.
These differences in metabolism likely account for the higher doses of ifosfamide required for equitoxic effects, the greater neurotoxicity
of ifosfamide, and the possible differences in antitumor spectrum of the two agents

Your Answer. Not Attempted


Correct Answer. a

Copyright 2014 Delhi Academy of Medical Sciences, All Rights Reserved. 72/84
(169). All are true about Ifosfamide except

a. It is nitrogen mustard

b. Metabolised by CYP3 A4 to form active metabolite

c. Chloracetaldehyde is active form

d. Less neurotoxic then cyclophosphamide

Solution. Ans 169: (d) Less neurotoxic then cyclophosphamide Ref Read the text below Sol
Ifosfamide is an oxazaphosphorine, similar to cyclophosphamide.
Cyclophosphamide has two chloroethyl groups on the exocyclic nitrogen atom, whereas one of the two-chloroethyl groups of ifosfamide
is on the cyclic phosphoamide nitrogen of the oxazaphosphorine ring.
Ifosfamide is activated in the liver by CYP3A4. The activation of ifosfamide proceeds more slowly, with greater production of
dechlorinated metabolites and chloroacetaldehyde.
These differences in metabolism likely account for the higher doses of ifosfamide required for equitoxic effects, the greater neurotoxicity
of ifosfamide, and the possible differences in antitumor spectrum of the two agents

Your Answer. d
Correct Answer. d

(170). Which of the following doesn't worsen angina

a. Sumatriptan

b. Oxyfedrine

c. Dipyridamole

d. Thyroxine

Solution. Ans 170: (b) Oxyfedrine Ref http://www.ncbi.nlm.nih.gov/pubmed/20326 Sol Mechanism of the antianginal action of
oxyfedrine
Oxyphedrine was found to depress the tonicity of coronary vessels, increase the coronary circulation volume, the oxygen uptake by the
heart and also to exert a positive inoand chronotropic action in test with urethan and cloralose anesthetized cats.
These effects were caused by stimulation of the betaadrenoreceptors of the heart and vessels.
In non-anesthetized cats oxyphedrine did not produce tachycardia. Furthermore, under certain conditions (when superimposed on a
preliminary administration of practolol- a beta-blocking agent) oxyphedrine can exert a direct inhibitory action on the myocardium.

Your Answer. a
Correct Answer. b

(171). Octreotide is used in all except

a. Insulinoma

b. Glucagonoma

c. Glioma

d. Carcinoids

Solution. Ans 171: (c) Glioma Ref Harrison 17th Edition Pg. 2228, 2225 Sol
Synthetic analogues of somatostatin (octreotide, lanreotide) are the most widely used agents to control the symptoms of patients with
carcinoid syndrome.
Octreotide controls symptoms in >80% of patients, including the diarrhea and flushing, and 70% of patients have a >50% decrease in
urinary 5HIAA excretion.
Long-acting somatostatin analogues such as octreotide are acutely effective in 40% of patients of insulinoma. However, octreotide needs
to be used with care because it inhibits growth hormone secretion and can alter plasma glucagon levels; therefore, in some patients it
can worsen the hypoglycemia
Long-acting somatostatin analogues such as octreotide or lanreotide improve the skin rash in 75% of patients of glucagonoma and may
improve the weight loss, pain, and diarrhea but usually do not improve the glucose intolerance.

Your Answer. b
Correct Answer. c

Copyright 2014 Delhi Academy of Medical Sciences, All Rights Reserved. 73/84
(172). All are true about exenatide except

a. Decreases glucagon secretion

b. It is GLP1 analogue

c. Used in type 1DM

d. Given subcutaneously

Solution. Ans 172: (c) Used in type 1DM Ref Read the text below Sol Exenatide
It is a synthetic analog of glucagon-like-polypeptide 1 (GLP-1). Exenatide is approved as an injectable, adjunctive therapy in persons with
type 2 diabetes treated with metformin or metformin plus sulfonylureas who still have suboptimal glycemic control. In clinical studies,
exenatide therapy was shown to have multiple actions such as potentiation of glucose-mediated insulin secretion, suppression of
postprandial glucagon release through as-yet unknown mechanisms, slowed gastric emptying, and a central loss of appetite. The
increased insulin secretion is speculated to be due in part to an increase in betacell mass. Exenatide is injected subcutaneously
within 60 minutes before a meal; therapy is initiated at 5 mcg twice daily, with a maximum dosage of 10 mcg twice daily. The major
adverse effects are nausea (about 44% of users) and vomiting and diarrhea. The nausea decreases with ongoing exenatide usage. Weight
loss is reported in some users, presumably because of the nausea and anorectic effects. A serious and, in some cases, fatal adverse effect
of exenatide is necrotizing and hemorrhagic pancreatitis.
SITAGLIPTIN
Sitagliptin is an inhibitor of dipeptidyl peptidase-4 (DPP-4), the enzyme that degrades incretin and other GLP-1-like molecules. Its major
action is to increase circulating levels of GLP-1 and GIP. This ultimately decreases postprandial glucose excursions by increasing
glucose-mediated insulin secretion and decreasing glucagon levels. Common adverse effects include nasopharyngitis, upper respiratory
infections, and headaches. Sitagliptin can be given as monotherapy or combined with metformin or Tzds

Your Answer. c
Correct Answer. c

(173). Inverse agonist of benzodiazepine receptors is

a. Phenobarbitone

b. Flumazenil

c. Beta-carboline

d. Gabapentin

Solution. Ans-173: (c) Beta carboline Ref: KDT-385 Sol:


-Carboline alkaloids are widespread in plants and animals, and frequently act as monoamine oxidase inhibitors (MAOI).
As components of the liana Banisteriopsis caapi, the -carbolines harmine, harmaline, and tetrahydroharmine play a pivotal role in the
pharmacology of the indigenous hallucinogenic drug ayahuasca by preventing the breakdown of dimethyltryptamine in the gut by
inhibiting monoamine oxidase, thus making it psychoactive upon oral administration.
Some -carbolines, notably tryptoline and pinoline, are formed naturally in the human body. The latter is implicated along with melatonin
in the role of the pineal gland in regulating the sleep-wake cycle
Inverse agonist of BZD receptor is carboline. Antagonists of BZD receptor is flumazenil.

Your Answer. c
Correct Answer. c

Copyright 2014 Delhi Academy of Medical Sciences, All Rights Reserved. 74/84
(174). Absolute contraindication to morphine use is

a. Left ventricular failure

b. Myocardial infarciton

c. Head injury

d. Terminal cancer pain

Solution. Ans-174: (c) Head injury Ref: Read the text below. Sol: Morphine is contraindicated in
Respiratory depression
Head injury
Hypersensitivity
Paralytic ileus
Delayed gastric emptying
Obstructive airway disease,
Acute hepatic disease
Mao inhibitor administration
Pregnancy
Lactation
Children.

Your Answer. c
Correct Answer. c

(175). Which of the following colloid solutions has proven thromboprophylactic effect?

a. Dextran

b. Gelatin preparation

c. Haemacceal

d. Hydroxyethyl starch

Solution. Ans-175: (a) Dextran Ref: KDT-584 Sol:


These agents are used commonly by microsurgeons to decrease vascular thrombosis.
The antithrombotic effect of dextran is mediated through its binding of erythrocytes, platelets, and vascular endothelium, increasing
their electronegativity and thus reducing erythrocyte aggregation and platelet adhesiveness.
Dextrans also reduce factor VIII-Ag Von Willebrand factor, thereby decreasing platelet function. Clots formed after administration of
dextrans are more easily lysed due to an altered thrombus structure (more evenly distributed platelets with coarser fibrin).
By inhibiting -2 antiplasmin, dextran serves as a plasminogen activator and therefore possesses thrombolytic feature
Dextran-40 has been tried in stroke and for prophylaxis of DVT and pulmonary infarction.

Your Answer. a
Correct Answer. a

(176). Hypercoagulability and dermal vascular necrosis are early appearing adverse effects of

a. Clopidogrel

b. Heparin

c. Warfain

d. Vitamin K

Solution. Ans-176: (c) Warfarin Ref: KDT-602 Sol:


Oral anticoagulants inhibit the activation of several clotting factors (II, VII, IX and X) as well as anti-clotting proteins (protein C and S).
First to disappear is protein C that can result in hypercoagulation resulting in dermal vascular necrosis

Your Answer. c
Correct Answer. c

Copyright 2014 Delhi Academy of Medical Sciences, All Rights Reserved. 75/84
(177). Which of the following is the clinical indication for the use of recombinant human erythropoietin?

a. Megaloblastic anemia

b. Hemolytic anemia

c. Anemia in patients of thalassemia

d. Anemia in chronic renal failure patients

Solution. Ans-177: (d) Anemia in chronic renal failure patients Ref: KDT-592 Sol:
Erythropoietin is available as a therapeutic agent produced by recombinant DNA technology in mammalian cell culture.
It is used in treating anemia resulting from chronic kidney disease and myelodysplasia, from the treatment of cancer (chemotherapy and
radiation).

Your Answer. d
Correct Answer. d

(178). The drug that does not result in theophylline toxicity is

a. Ciprofloxacin

b. Amoxicillin

c. Erythromycin

d. Cimetidine

Solution. Ans-178: (b) Amoxycillin Ref: KDT-221 Sol:


Theophylline is metabolized by microsomal enzymes.
Enzyme inhibitors like ciprofloxacin, erythromycin and cimetidine decrease the metabolism of theophylline and can result in toxicity.

Your Answer. b
Correct Answer. b

(179). A patient undergoing cancer chemotherapy is vomiting frequently. A drug that might help in this situation is

a. Bromocriptine

b. Ketanserin

c. Loratidine

d. Ondansetron

Solution. Ans-179: (d) Ondansetron Ref: KDT- 646 Sol:


The 5-HT3 receptor antagonists are the primary drugs used to treat and prevent chemotherapy-induced nausea and vomiting (CINV).
A common use case is to give them intravenously about 30 minutes before commencement of a chemotherapy treatment.
Ondansetron is also effective in controlling post-operative nausea and vomiting (PONV) and post-radiation nausea and vomiting, and is a
possible therapy for nausea and vomiting due to acute or chronic medical illness or acute gastroenteritis.

Your Answer. d
Correct Answer. d

Copyright 2014 Delhi Academy of Medical Sciences, All Rights Reserved. 76/84
(180). The most important channel of elimination of digoxin is

a. Glomerular filtration

b. Tubular secretion

c. Hepatic metabolism

d. Excretion in bile

Solution. Ans-180: (a) Glomerular filtration Ref: KDT - 497 Sol:


Digoxin is primarily eliminated unchanged by glomerular filtration whereas digitoxin is eliminated by hepatic metabolism.

Your Answer. c
Correct Answer. a

(181). Prolonged use of hydralazine for the treatment of hypertension is likely to cause

a. Gynaecomastia

b. Thrombocytopenia

c. Hemolytic anemia

d. Lupus erythematosis

Solution. Ans-181: (d) Lupus erythematosis Ref: KDT-548 Sol: Common side-effects include:
Diarrhea
Compensatory tachycardia due to baroreceptor reflex ->Angina
Headache
Loss of appetite
Nausea or vomiting
Depression
Pounding heartbeat
Drug-Induced Lupus Erythematosus
ANCA-associated Vasculitis - Generally MPO-ANCA positive
Patients given hydralazine over a period of six months may develo
Hydralazine is differentially acetylated by fast and slow acetylator phenotypes, hence incidence of lupus-like disease in slow acetylators

Your Answer. d
Correct Answer. d

(182). Which of the following diuretics can result in metabolic acidosis ?

a. Indapamide

b. Furosemide

c. Hydrochlorthiazide

d. Acetazolamide

Solution. Ans-182: (d) Acetazolamide Ref: KDT-569 Sol:


Carbonic anhydrase inhibitors decrease the excretion of H+ and increase the excretion of HCO3- in the urine. This may result in
metabolic acidosis and urinary alkalosis.
Thiazides and loop diuretics result in metabolic alkalosis

Your Answer. b
Correct Answer. d

Copyright 2014 Delhi Academy of Medical Sciences, All Rights Reserved. 77/84
(183). Which of the following agents is useful for the oral treatment of both pituitary as well as renal diabetes insipidus?

a. Vasopressin

b. Hydrochlorthiazide

c. Chlorpropamide

d. Carbamazepine

Solution. Ans-183: (b) Hydrochlorthiazide Ref: KDT-578 Sol:


Central DI and gestational DI respond to desmopressin. Carbamazepine, an anti-convulsive medication, has also had some success in
this type of DI. Also gestational DI tends to abate on its own 4 to 6 weeks following labour, though some women may develop it again in
subsequent pregnancies. In dipsogenic DI, desmopressin is not usually an option.
Desmopressin will be ineffective in nephrogenic DI. Instead, the diuretic hydrochlorothiazide (a thiazide diuretic) or indomethacin can
improve nephrogenic diabetes insipidus. Thiazide diuretics are sometimes combined with amiloride to prevent hypokalemia. Thiazides
are useful in the treatment of central as well as nephrogenic DI.
Lithium-induced nephrogenic DI may be effectively managed with the administration of amiloride, a potassium-sparing diuretic often
used in conjunction with thiazide or loop diuretics. Clinicians have been aware of lithium toxicity for many years and traditionally have
administered thiazide diuretics for lithium-induced polyuria and nephrogenic diabetes insipidus. However, recently amiloride has been
shown to be a successful treatment for this condition
Vasopressin, chlorpropamide and carbamazepine are useful only in central DI.

Your Answer. b
Correct Answer. b

(184). Which of the following is not an indication for oxytocin

a. Spontaneous premature labour

b. Post partum haemorrhage

c. Uterine inertia

d. Breast engorgement due to inefficient milk ejection reflex

Solution. Ans-184: (a) Spontaneous premature labour Ref: KDT-328 Sol: INDICATIONS Antepartum
Oxytocin) is indicated for the initiation or improvement of uterine contractions, where this is desirable and considered suitable, in order
to achieve early vaginal delivery for fetal or maternal reasons.
It is indicated for (1) induction of labor in patients with a medical indication for the initiation of labor, such as Rh problems, maternal
diabetes, pre-eclampsia at or near term, when delivery is in the best interest of mother and fetus or when membranes are prematurely
ruptured and delivery is indicated; (2) stimulation or reinforcement of labor, as in selected cases of uterine inertia; (3) as adjunctive
therapy in the management of incomplete or inevitable abortion.
In the first trimester, curettage is generally considered primary therapy. In the second trimester abortion, oxytocin infusion will often be
successful in emptying the uterus. Other means of therapy, however, may be required in such cases.
Postpartum Oxytocin injection is indicated to produce uterine contractions during the third stage of labor and to control postpartum
bleeding or hemorrhage.
Spotaneous premature labour is an indication for use of tocolytics and not oxytocin.

Your Answer. a
Correct Answer. a

Copyright 2014 Delhi Academy of Medical Sciences, All Rights Reserved. 78/84
(185). The antithyroid drug with the most rapid onset of antithyroid action is

a. I131

b. Sodium iodide

c. Methimazole

d. Propylthiouracil

Solution. Ans-185: (b) Sodium iodide Ref: KDT-251 Sol:


Iodides inhibit the secretion of thyroid hormones in the circulation and therefore are the fastest acting antithyroid drugs.
I131 takes more than 3 weeks to manifest their action.
Thyroid peroxidase inhibitors are delayed acting because their action manifests only when already stored pool of thyroid hormones is
utilized.

Your Answer. b
Correct Answer. b

(186). Which of the following disorder is not aggravated by corticosteroid therapy?

a. Congenital adrenal hyperplasia

b. Diabetes mellitus

c. Hypertension

d. Peptic ulcer

Solution. Ans-186: (a) Congenital adrenal hyperplasia Ref: KDT-283 Sol:


Corticosteroids are used for the management of congenital adrenal hyperplasia.
Due to retention of Na+ and water, corticosteroids can worsen the hypertension.
Corticosteroids can result in hyperglycemia and thus may vitigate the control of blood glucose in diabetics.
By inhibiting the production of gastroprotective prostaglandins, corticosteroids increase the risk of peptic ulcer disease.

Your Answer. a
Correct Answer. a

(187). Antieptic drug of choice in child treated with ketogenic diet?

a. Carbamazipine

b. Phentoin

c. Valproate

d. Lamotrigine

Solution. Ans-187: (c) Valproate Ref.: Internet www.epilepsy.com Sol :


Ketogenic diet and medication combinations.
The ketogenic diet is commonly used in conjunction with one or more anticonvulsant medication, and more is being learned about the
risks and benefits of these combinations. To begin with, the ketogenic diet does not seem to change the blood (plasma) levels of common
anticonvulsant medications in any significant way.
Investigators have measured plasma levels both before and after diet initiation and found no significant differences.
It seems that you dont need to decrease or increase any medications when starting the diet to keep blood levels the same.
Regarding tolerability, valproic acid, topiramate and zonisamide have garnered the most concern, as the side effects of the ketogenic
diet and these medications overlap.

Your Answer. Not Attempted


Correct Answer. c

Copyright 2014 Delhi Academy of Medical Sciences, All Rights Reserved. 79/84
(188). Phenobarbitone in mother causes all in baby except :

a. Hypoglycemia

b. Ataxia

c. Neonatal drowsiness

d. Vitamin K deficiency

Solution. Ans-188: (a) Hypoglycemia Ref.: Martindale 36th ed.P.492 Sol :


The most frequent adverse effect associated with Phenobarbital is sedation.
Prolonged use can occasionally result in folate deficiency; rarely, megaloblastic anaemia has been reported. There is some evidence that
phenobarbital interferes with vitamin D metabolism.
At high doses nystagmus and ataxia may occur and the typical barbiturate-induced respiratory depression may become severe.
Neonatal drug dependence and symptoms resembling vitamin K deficiency have been reported in infants born to mothers who received
phenobarbital during pregnancy.
Congenital malformations have been reported in children of women who received phenobarbital during pregnancy but the causal role of
the drug is a matter of some debate.

Your Answer. d
Correct Answer. a

(189). Entry of new drug into the market :

a. Phase 1

b. Phase 2

c. Phase 3

d. Phase 4

Solution. Ans-189: (c) Phase 3 Ref.: KDT - P.77 Sol :


Phase 3 is also known as therapeutic confirmation/comparison.
Generally these are randomized double blind comparative trials conducted on a larger patient population (500-3000) by several
physicians at many centres.
The aim is to establish the value of the drug in relation to existing therapy.
Safety, tolerability and possible drug interactions are assessed on a wider scale, while additional pharmacokinetic data may be obtained.
A new drug application (NDA) is submitted to the licencing authority, who if convinced give marketing permission.
Phase 2 is known as Therapeutic Exploratory and carried out in patients.
Phase 1 trials are done on healthy volunteers
IND is the investigational new drug which is filed before starting trial on human beings.

Your Answer. c
Correct Answer. c

(190). What is the mechanism of action of Apixaban ?

a. It is a factor X a inhibitor

b. It is thrombin inhibitor

c. it is a anti thrombin inhibitor

d. It is anti platelet inhibitor

Solution. Ans-190: (d) It is a factor X a inhibitor Ref: Read the text below Sol:
Apixaban (INN, trade name Eliquis) is an anticoagulant for the prevention of venous thromboembolism and venous thromboembolic
events.
It is a direct factor Xa inhibitor. Apixaban has been available in Europe since May 2011 and was approved for preventing venous
thromboembolism after elective hip or knee replacement.
The FDA approved apixaban in December 2012 with an indication of reducing the risk of stroke and dangerous blood clots (systemic
embolism) in patients with atrial fibrillation that is not caused by a heart valve problem

Your Answer. a
Correct Answer. d

Copyright 2014 Delhi Academy of Medical Sciences, All Rights Reserved. 80/84
(191). Clofazimine therapy can cause all except

a. Skin

b. Icthyosis

c. Subacute

d. Peripheral

Solution. Ans-191: (d) Peripheral neuropathy Ref: Read the text below Sol:
Nimodipine (marketed by Bayer as Nimotop) is a dihydropyridine calcium channel blocker originally developed for the treatment of high
blood pressure.
It is not frequently used for this indication, but has shown good results in preventing a major complication of subarachnoid hemorrhage
(a form of cerebral hemorrhage) termed vasospasm; this is now the main use of nimodipine.
Nimodipine binds specifically to L-type voltage-gated calcium channels. There are numerous theories about its mechanism in preventing
vasospasm, but none are conclusive.

Your Answer. Not Attempted


Correct Answer. d

(192). Which of the following is a direct rennin inhibitor?

a. Aliskrein

b. Losartan

c. Perindopril

d. Vernakalant

Solution. Ans-192: (a) Aliskrein Ref.: Goodman Gilmans - 799 Sol :


DRIs are a novel class of antihypertensive drugs that inhibit the RAS at its origin.
Angiotensinogen is the only specific substrate for rennin, and its conversion to Angl presents a rate limiting step for the generation of
downstream components of the RAS.
Aliskiren is a low-molecular weight non-peptide that is a potent competitive inhibitor of renin. It binds the active site of rennin of block
conversion of angiotensinogen to Angl, thus reducing the consequent production of AngII.
Aliskiren has a 10,000-fold higher affinity to rennin than to any other aspartic peptidases.
The t is 20-45 hours.
Plasma protein binding is 50% and is independent of concentration

Your Answer. a
Correct Answer. a

(193). The following drugs enhance fibrinolysis except

a. Phenformin

b. Urokinase

c. Streptokinase

d. Alteplase

Solution. Ans-193: (a) Phenformin Ref: Read the text below Sol :
Phenformin, along with buformin and metformin, inhibits the growth and development of cancer.
The anticancer property of these drugs may be due to their ability to disrupt the Warburg effect and revert the cytosolic glycolysis
characteristic of cancer cells to normal oxidation of pyruvate by the mitochondria.
Metformin reduces liver glucose production in diabetics and disrupts the Warburg effect in cancer by AMPK activation and inhibition of
the mTor pathway

Your Answer. a
Correct Answer. a

Copyright 2014 Delhi Academy of Medical Sciences, All Rights Reserved. 81/84
(194 All of the following medications used for bronchial asthma act as
). bronchodilators except

a. Albuterol

b. Fluticasone propionate

c. Theophylline

d. Toflimilast

Solution. Ans-194: (b) Fluticasone propionate Ref.: Read the text below
Sol : Fluticasone propionate.

Your Answer. b
Correct Answer. b

(195). According to a recent FDA recommendation, the dosage of paracetamol in a single tablet should not exceed

a. 325 mg

b. 450 mg

c. 500 mg

d. 650 mg

Solution. Ans-195: (a) 325 mg Ref.: Read the text below Sol :
According to a recent FDA recommendation issued in January 2014, the dosage of paracetamol in a single tablet should not exceed 325
mg.

Your Answer. d
Correct Answer. a

(196). The incidence of overt hypothyroidism due to lithium is

a. 7 10%

b. ~ 20%

c. ~ 60%

d. 15 70%

Solution. Ans-196: (a) 7 10% Ref.: Read the text below Sol : Side effects of Lithium
Hypothyroidism 7 10%
Reversible T wave flattening ~ 20%
Polyuria / DI ~ 60%
Fine tremors 15 70%

Your Answer. d
Correct Answer. a

Copyright 2014 Delhi Academy of Medical Sciences, All Rights Reserved. 82/84
(197). All of the following diuretics act from the luminal side of the nephron except

a. Bumetanide

b. Eplerenone

c. Amiloride

d. Acetazolamide

Solution. Ans-197: (b) Eplerenone Ref.: Read the text below Sol : Eplerenone. Aldosterone receptor antagonists act from the basolateral
side.

Your Answer. Not Attempted


Correct Answer. b

(198). The father of pharmacology is

a. Rudolph Buchheim

b. Oswald Schmiedeberg

c. Paul Ehrlich

d. Gerhard Domagk

Solution. Ans-198: (b) Oswald Schmiedeberg Ref.: Read the text below Sol : Oswald Schmiedeberg. He gave many fundamental concepts
of pharmacology.
Rudolph Buchheim opened the first institute for research in pharmacology in Germany.
Paul Ehrlich developed arsphenamine and neoarsphenamine for syphilis; and atoxyl for sleeping sickness. He also coined the term
chemotherapy
Gerhard Domagk used prontosil for treatment of streptococcal infections. Sulphonamides were later developed from these dyes.

Your Answer. a
Correct Answer. b

(199). The biochemical mechanism of action of digitalis is associated with

a. A decrease in calcium uptake by sarcoplasmic reticulum

b. An increase in ATP synthesis

c. An increase in the systolic extracellular calcium levels

d. A block of sodium/calcium exchange

Solution. Ans-199: (d) A block of sodium/calcium exchange Ref.: Read the text below Sol : A block of sodium/calcium exchange. Digitalis
causes inhibition of Na+-K+-ATPase pump leading to secondary
Increase in the systolic intracellular calcium levels
Inhibition of sodium/calcium exchange and
Increase in calcium uptake by sarcoplasmic reticulum

Your Answer. a
Correct Answer. d

Copyright 2014 Delhi Academy of Medical Sciences, All Rights Reserved. 83/84
(200). Which of the following pairs of drug and its classification is correct?

a. Tigecycline Tetracycline

b. Nalidixic acid Fluoroquinolone

c. Dalbavancin Oxazolidinedione

d. Spiramycin Macrolide

Solution. Ans-200: (d) Spiramycin Macrolide Ref.: Read the text below Sol : Spiramycin Macrolide
Tigecycline is a glycylcycline.
Nalidixic acid is a quinolone.
Dalbavancin is a glycopeptide.

Your Answer. b
Correct Answer. d

Test Answer
1.(c) 2.(d) 3.(b) 4.(d) 5.(c) 6.(b) 7.(c) 8.(b) 9.(b) 10.(a)

11.(d) 12.(d) 13.(c) 14.(a) 15.(b) 16.(b) 17.(c) 18.(b) 19.(a) 20.(b)

21.(b) 22.(a) 23.(a) 24.(c) 25.(d) 26.(d) 27.(b) 28.(c) 29.(a) 30.(b)

31.(c) 32.(d) 33.(b) 34.(b) 35.(a) 36.(b) 37.(c) 38.(c) 39.(d) 40.(d)

41.(a) 42.(d) 43.(b) 44.(d) 45.(a) 46.(b) 47.(a) 48.(a) 49.(c) 50.(a)

51.(a) 52.(b) 53.(c) 54.(d) 55.(d) 56.(b) 57.(d) 58.(c) 59.(c) 60.(c)

61.(a) 62.(c) 63.(d) 64.(a) 65.(d) 66.(d) 67.(d) 68.(b) 69.(d) 70.(c)

71.(d) 72.(a) 73.(c) 74.(b) 75.(a) 76.(c) 77.(b) 78.(d) 79.(b) 80.(c)

81.(a) 82.(d) 83.(d) 84.(a) 85.(c) 86.(c) 87.(d) 88.(a) 89.(c) 90.(a)

91.(d) 92.(d) 93.(c) 94.(c) 95.(b) 96.(c) 97.(d) 98.(d) 99.(d) 100.(a)

101.(d) 102.(d) 103.(c) 104.(d) 105.(d) 106.(c) 107.(b) 108.(d) 109.(d) 110.(a)

111.(b) 112.(d) 113.(d) 114.(d) 115.(d) 116.(a) 117.(a) 118.(b) 119.(b) 120.(d)

121.(c) 122.(b) 123.(a) 124.(d) 125.(a) 126.(a) 127.(d) 128.(b) 129.(b) 130.(c)

131.(d) 132.(c) 133.(c) 134.(d) 135.(d) 136.(c) 137.(c) 138.(d) 139.(d) 140.(b)

141.(a) 142.(a) 143.(d) 144.(d) 145.(b) 146.(c) 147.(d) 148.(d) 149.(c) 150.(d)

151.(b) 152.(b) 153.(b) 154.(c) 155.(b) 156.(c) 157.(d) 158.(d) 159.(b) 160.(b)

161.(d) 162.(c) 163.(a) 164.(a) 165.(d) 166.(a) 167.(b) 168.(a) 169.(d) 170.(b)

171.(c) 172.(c) 173.(c) 174.(c) 175.(a) 176.(c) 177.(d) 178.(b) 179.(d) 180.(a)

181.(d) 182.(d) 183.(b) 184.(a) 185.(b) 186.(a) 187.(c) 188.(a) 189.(c) 190.(d)

191.(d) 192.(a) 193.(a) 194.(b) 195.(a) 196.(a) 197.(b) 198.(b) 199.(d) 200.(d)

Copyright 2014 Delhi Academy of Medical Sciences, All Rights Reserved. 84/84

Anda mungkin juga menyukai